Location via proxy:   [ UP ]  
[Report a bug]   [Manage cookies]                

T5 Test

Download as pdf or txt
Download as pdf or txt
You are on page 1of 100

.

IASBABA’S PRELIMS TEST SERIES (AIPTS) 2024 – TEST 5 (MEDIEVAL HISTORY)

Q.1) With reference to the Tripartite a) 1 and 2 only


Struggle of the 8th century AD, consider the
b) 3 only
following statements:
c) 1 only
1. It was struggle between Palas in the
East, Pratihara in the North and d) All them are from Gurjara-Pratihara
Rashtrakutas in the Deccan. Dynasty
2. The main cause for this struggle was
the desire to possess the city of
Kannauj which was a symbol of Q.3) Consider the following abstract:
sovereignty at that time.
“The notable figure was a Sanskrit poet,
3. The tripartite struggle for Kannauj dramatist, and critic who resided in the
spanned almost two centuries, court of Mahendrapala I, ruler of the
resulting in Druva II's triumph and Gurjara-Pratihara dynasty. He authored the
the subsequent establishment of Kavyamimamsa, a handbook that provides
Kannauj as the capital of the practical guidance for poets by outlining the
Rashtrakutas empire. fundamental elements and structure of a
quality poem. The individual is renowned
How many of the statements given above for their production of the play
is/are correct? 'Karpuramanjari.' ”
a) Only one Select the correct poet with whom these
achievements are associated with:
b) Only Two
a) Rajasekhara
c) All Three
b) Sandhyakara Nandi
d) None
c) Vajrasattva

d) Brindaban Das
Q.2) Which of the following rulers does not
belong to the Gurjara-Pratihara Dynasty?

1. Vatsaraja Q.4) With reference to the Pala dynasty,


consider the following statements:
2. Mihir Bhoja
1. The Pala Empire was established by
3. Mahendrapala I
a king who was chosen through an
Select the correct answer by using the code electoral process.
below:
.
IASBABA’S PRELIMS TEST SERIES (AIPTS) 2024 – TEST 5 (MEDIEVAL HISTORY)

2. The Pala rulers embraced the role of Q.6) With reference to Rashtrakuta dynasty,
patrons for Buddhism. consider the following statements:

3. The Nalanda University was founded 1. It was founded by Dantidurga the


by the Pala rulers feudatory of the Chalukyan king
Kirtivarman II.
How many of the statements given above
is/are correct? 2. Amoghavarsha was known to have
authored texts both in Kannada and
a) Only one
Sanskrit.
b) Only Two
3. The Arab traveler Sulaiman
c) Only Three compared Amoghavarsha to
Emperor Ashoka and King
d) None Vikramaditya of the Gupta dynasty.

How many of the statements given above


is/are correct?
Q.5) Which of the following statements
is/are correct about Rashtrakuta dynasty? a) Only one
1. The Rashtrakuta rulers were tolerant b) Only Two
in their religious views.
c) Only Three
2. The famous rock-cut temple of Siva
at Ellora was built by the d) None
Rashtrakuta king Amoghavarsha.

3. The revenue officers of the Deccan


Q.7) Jinasena, the renowned Jain scholar
were known as nad-gavundas.
and author of Adipurana and Mahapurana,
How many of the statements given above served at the court of:
is/are correct?
a) MihiraBhoja of Prathihara
a) Only one
b) Dharmapala of Pala
b) Only Two
c) Amoghavarsha I of Rashtrakuta
c) All Three
d) Rajendra I of Chola
d) None
.
IASBABA’S PRELIMS TEST SERIES (AIPTS) 2024 – TEST 5 (MEDIEVAL HISTORY)

Q.8) Which of the following were the causes 4. The administration in the villages
for the failure of native Kingdoms during was under the elders who were
early medieval period? elected through lottery.

1. Mutual conflicts among Rajput. How many of the statements given above
is/are correct?
2. Dependency on quick-moving
cavalry by Rajput. a) Only one

3. Participation of all major classes in b) Only Two


wars.
c) Only Three
4. Weak organization and inefficient
d) All four statements are correct
leadership.

How many of the statements given above


is/are correct? Q.10) The famous ‘Uttaramerur Inscription ’
belongs to the reign of:
a) Only one
a) Vijayalaya
b) Only Two
b) Parantaka I
c) Only Three
c) UttamaChola
d) All Four Statements are correct
d) Rajaraja I

Q.9) Consider the following statements


regarding the general administration under Q.11) Who held the position of ruler in the
the Chola Empire: Pala dynasty during Rajendra Chola's
expedition to the Gangetic Valley?
1. The Council of Ministers had the real
power over and above the king. a) Dharmapala
2. The basic unit of administration was b) Devapala
Nadu.
c) Mahipala I
3. The Governors of the provinces
were selected from the local people d) Ramapala
through elections.
.
IASBABA’S PRELIMS TEST SERIES (AIPTS) 2024 – TEST 5 (MEDIEVAL HISTORY)

Q.12) Which of the following statements How many of the statements given above
is/are correct? is/are correct?

1. Kampaharesvara temple at a) Only one


Tribhuvanam is an example of the
b) Only Two
sangam age Chola temple.
c) All Three
2. The inscription at Thirubhuvanai
provides the details of the colleges d) None
that existed in the Chola territory.

3. The Cholas are known for importing


horses from Arabia. Q.14) Consider the following abstract:

Select the correct answer by using the code “He was a great conqueror, extending his
below: kingdom to as far as parts of Sri Lanka.
Founded the city of
a) 1 and 2 only Gangaikondacholapuramand constructed
the famous Rajesvaram temple in that city.
b) 1 and 3 only
The river Tungabadhra became his northern
c) 2 and 3 only boundary. He assumed the title of
KadaramKondan after his naval expedition
d) 1, 2 and 3
to Kadaram or Sri Vijaya.’’

Select the correct ruler with whom these


Q.13) With reference to the administration achievements are associated with:
under the Chola, consider the following
a) Kulottungachola II
statements:
b) Rajarajachola I
1. The residential portions of a village
under the Cholas were exempted c) Bhaskararavivarman
from taxes.
d) Rajendrachola I
2. The knowledge of Vedas was
mandatory for becoming a village
council member under the Cholas. Q.15) With reference to the history of
medieval India which of the following
3. During the Chola reign, both the
branch does Prithiviraj III belongs to?
Malabar and Coromandel coasts
were under their control. a) Chauhans of Shakambari

b) Chauhans of Ajmer
.
IASBABA’S PRELIMS TEST SERIES (AIPTS) 2024 – TEST 5 (MEDIEVAL HISTORY)

c) Chauhans of Ranthambor Q.18) Consider the following statements


about Paramara king Bhoja?
d) Chauhans of Jalore
1. He was the most celebrated ruler of
the Paramaras dynasty and had a
Q.16) Consider the following statement capital in kannauj.
regarding the Chandellas of Bundelkhand:
2. He founded the city of Bhojpur and
1. Initially, they were feudatories of constructed three dams in that area.
the Gurjara-Pratiharas of Kannauj.
3. He was the author of many works on
2. Dhang deva was the first a variety of subjects like medicine,
Independent Chandella king. astronomy, religion, grammar,
architecture, etc.
3. They are well known for the Vesara-
style Art and architecture. How many of the statements given above
is/are correct?
Which of the above given statement is/are
correct? a) Only one

a) 1 and 2 only b) Only Two

b) 1 and 3 only c) All Three

c) 2 and 3 only d) None

d) 1, 2 and 3
Q.19) Consider the following statement
about the Kumara Pala, the king of
Q.17) Which among the following Dynasty Chalukyas of Gujarat:
built the famous temple ‘Kandariya
Mahadeva temple’? 1. He was born into a family that
followed the Shaivite tradition but
a) Chola Dynasty later he made a decision to convert
to Jainism.
b) Pala Dynasty
2. During his reign, Jainism gained
c) Chandela Dynasty
prominence in Gujarat.
d) Rashtrakutra Dynasty
3. Due to his religious beliefs, Kumara
Pala built several temples, including
Brahminical temples.
.
IASBABA’S PRELIMS TEST SERIES (AIPTS) 2024 – TEST 5 (MEDIEVAL HISTORY)

Which of the above given statements is/are Q.21) Who were the primary combatants in
correct? the Peshawar Battle of 1002 AD?

a) 1 and 2 only a) The Mughal Empire and the Pashtun


tribes
b) 1 and 3 only
b) The Ghaznavid Empire and the
c) 2 and 3 only
Hindu Shahi Kingdom
d) 1, 2 and 3
c) The Ghurid Empire and the
Khwarazmian Empire

Q.20) With reference to the Rani-Ki-Vav d) The Abbasid Caliphate and the
(Queens step well), Consider the following Samanid Empire
statements:

1. The famous Rani–ki-vav (Queens


Q.22) Consider the following pairs of
step well) was built by Rani
famous Rajput kings and their dynasty:
Udaymati, in memory of her
husband, King Bhimdev I. (Kings) (Dynasty)

2. The stepwell’s walls feature 1. Anangapala II: The Tomaras


sculptures of Hindu deities and Lord
2. Jaichandra: The
Vishnu's life scenes.
Ghadavalas
3. Rani-ki-vav, a stunning and
3. Avantivarman: The Utpala
historically important stepwell was
designated a UNESCO World How many of the following pairs is/are
Heritage site. correctly matched?
Which of the above given statements is/are a) Only one pair
correct?
b) Only two pairs
a) 1 and 2 only
c) Only three pairs
b) 1 and 3 only
d) None of the pairs are correctly
c) 2 and 3 only matched
d) 1, 2 and 3
.
.
.
.
IASBABA’S PRELIMS TEST SERIES (AIPTS) 2024 – TEST 5 (MEDIEVAL HISTORY)

a) Both Statement-I and Statement-II b) 1, 2 and 4 only


are correct and Statement-II is the
c) 1, 2 and 3 only
correct explanation for Statement-I.
d) 1, 2, 3 and 4
b) Both Statement-I and Statement-II
are correct and Statement-II is not
the correct explanation for
Statement-I. Q.35) Consider the following statements:

c) Statement-I is correct and but 1. Tahkik-i-Hind gives account of


Statement-II is incorrect. India’s scientific traditions.

d) Statement-I is incorrect but 2. Tarikh-i-Firozshahi exclusively deals


Statement-II is correct. with reign of Firozshah.

3. Fatawa-i-Jahandari defines ethics of


administration.
Q.33) Who of the following was the author
of the well-known work “Padmavat”? Which of the statements given above is/are
correct?
a) Abdul QadirBadayuni
a) 1 and 2 only
b) AbulFazl
b) 1 and 3 only
c) Abbas Sarwani
c) 2 and 3 only
d) Malik Muhammad Jayasi
d) 1, 2 and 3

Q.34) Which of the following towns were


established by Firoz Shah Tughlaq? Q.36) Consider the following statements
with reference to Medieval India:
1. Fatehabad
1. Architecture underwent significant
2. Hissar changes in the age of Delhi
sultanate, one of them being, usage
3. Faridabad
of Limestone cement, to build
4. Jaunpur stronger and larger constructions.

Select the correct answer by using the code 2. The Hindu rulers in the medieval age
given below: turned out to be the biggest patrons
for temple construction, because
a) 1, 3 and 4 only
they were also meant to
.
IASBABA’S PRELIMS TEST SERIES (AIPTS) 2024 – TEST 5 (MEDIEVAL HISTORY)

demonstrate the power and wealth c) Statement-I is correct and but


of the king. Statement-II is incorrect.

3. Jama Masjid in Delhi was built by d) Statement-I is incorrect but


Jahangir. Statement-II is correct.

4. The Chaarbagh gardens are


associated with Mughals.
Q.38) Consider the following statements
Which of the above statements are correct? about the monetary reforms of
Muhammad-Bin-Tughlaq:
a) 1 and 3 only
Statement-I: He introduced a copper coin
b) 1, 2 and 4 only
with the same value as the silver tanka.
c) 2, 3 and 4 only
Statement-II: Token currency was
d) 1, 2 and 3 only introduced for the first time ever in the
world history.

Which one of the following is correct in


Q.37) Consider the following statements
respect of the above statements?
with respect to the Amara-Nayaka System:
a) Both Statement-I and Statement-II
Statement-I: The Amara-Nayaka System
are correct and Statement-II is the
was influenced by the iqta system of the
correct explanation for Statement-I.
Delhi Sultanate.
b) Both Statement-I and Statement-II
Statement-II: Amara-Nayakas were military are correct and Statement-II is not
commanders, whose main work was to the correct explanation for
collect revenue. Statement-I.
Which one of the following is correct in c) Statement-I is correct and but
respect of the above statements? Statement-II is incorrect.
a) Both Statement-I and Statement-II d) Statement-I is incorrect but
are correct and Statement-II is the Statement-II is correct.
correct explanation for Statement-I.

b) Both Statement-I and Statement-II


are correct and Statement-II is not
the correct explanation for
Statement-I.
.
IASBABA’S PRELIMS TEST SERIES (AIPTS) 2024 – TEST 5 (MEDIEVAL HISTORY)

Q.39) With reference to the Muhammad a) 1 and 2 only


bin Tughlaq, Consider the following
b) 2 only
statements:
c) 3 only
1. He was tolerant in religious matters.
d) 2 and 3 only
2. The historian, Ziauddin Barani was
contemporary of Muhammed Bin-
Tughlaq.
Q.41) Which of the following literary
3. He withdrew diplomatic relations in compositions serve as sources of the history
order to prevent his empire from of the Vijayanagar Empire?
foreign invasion.
1. Ratnavali
Which of the statements given above is/are
correct? 2. Manucharitam

a) 1 and 2 only 3. Kavyamimansa

b) 1 and 3 only 4. Amukthamalyada

c) 2 and 3 only Select the correct answer using the code


given below:
d) 1, 2 and 3
a) 1, 2 and 4 only

b) 2, 3 and 4 only
Q.40) Which of the following statements
is/are correct about Vijayanagara Kingdom? c) 2 and 4 only

1. The Kudirai Chettis were a local d) 1 and 3 only


community of cloth merchants.

2. A key attraction of the Vijayanagar


Q.42) Which of the following statements
kingdom was the inclusion of
are correct?
agricultural tracts within the
fortified walls. 1. Achyuta Devaraya built a new city,
called Nagalapuram.
3. The Hiriya canal was constructed
under the Tuluva dynasty. 2. Krishnadevaraya is known to have
authored texts both in Telugu and
Select the correct answer by using the code
Sanskrit.
given below:
.
IASBABA’S PRELIMS TEST SERIES (AIPTS) 2024 – TEST 5 (MEDIEVAL HISTORY)

3. The presence of slavery in the Q.44) With reference to the Bahmani


Vijayanagar kingdom has been kingdom of medieval India, the term
attested by Nicolo de Conti. "Bargirs" refers to which of the following?

Select the correct answer by using the code a) Cavalrymen


given below:
b) Land grant made to military officers
a) 1 and 2 only
c) Land revenue
b) 1 and 3 only
d) Waterwheel used in the irrigation of
c) 2 and 3 only land

d) 1, 2 and 3

Q.45) During the Bahmani kingdom, the


term "Afaqis" referred to:
Q.43) With reference to Firuz Shah
Bahmani, Consider the following a) Those who are Involved in the
statements: minting of gold coins issued by the
kings.
1. He encouraged the pursuit of
astronomy and built an observatory b) The newcomers who were
near Daulatabad. foreigners.

2. He respected the tenets of all c) The huge land owners in the


religions. countryside.

3. The Bahmani capital was transferred d) Those who headed the trade unions
to Bidar during his reign. or the guilds and acted as the
bankers and the financers.
Which of the above given statements is/are
correct?

a) 1 and 2 only Q.46) With reference to the Vijayanagara


empire, Consider the following statements:
b) 1 and 3 only
1. He was a dominant ruler of the
c) 2 and 3 only
Sangam dynasty.
d) 1, 2 and 3
2. He constructed the Hazara
Ramaswamy temple in Hampi.
.
IASBABA’S PRELIMS TEST SERIES (AIPTS) 2024 – TEST 5 (MEDIEVAL HISTORY)

3. Abdur Razzak visited Vijayanagara a) Only one


during his reign and praised it as
b) Only Two
unparalleled in the world.
c) All Three
Based on the above statements, which of
the following ruler matches the description d) None
provided?

a) Devaraya II
Q.48) Consider the following statements
b) Srikrishna deva raya with reference to the status of women
during the Vijayanagara Empire:
c) Harihara I
1. The Vijayanagara Empire saw the
d) Bukkaraya I
involvement of women in various
fields including warfare and
wrestling.
Q.47) With reference to Nayankara system
of Vijayanagara Empire, consider the 2. The Devadasi system involved
following statements: women being dedicated to temples
and often forced into sexual
1. Under this system, the feudatories
servitude for the pleasure of men in
of the empire were granted partial
the royal court and visiting the
independence, with Nayankaras
temples.
receiving land in exchange for their
military service to the king. 3. The practice of Sati was abolished
during the court of Krishnadevaraya.
2. They were exempted from paying an
annual tribute to the king but were How many of the statements given above
obligated to provide a specific is/are correct?
number of soldiers, horses, and
a) Only one
elephants for the defense of the
empire. b) Only Two
3. The Nayankaras had to take the c) All Three
"sovereign oath" at the king's court,
which was a symbolic act to d) None
demonstrate their loyalty to the
empire.

How many of the statements given above


is/are correct?
.
IASBABA’S PRELIMS TEST SERIES (AIPTS) 2024 – TEST 5 (MEDIEVAL HISTORY)

Q.49) With reference to the Tax system of 2. Private individuals commissioned by


Vijayanagara Empire, consider the following the emperor were also able to mint
statements: coins, however coins issued by
private individuals were in use only
1. In addition to land revenue, tolls
in certain areas.
were collected from festivals held in
cities and towns. Which of the above given statements is/are
correct?
2. During times of war, the
government would impose special a) 1 only
taxes on both the subjects and
b) 2 only
feudal lords.
c) Both 1 and 2
3. The government also levied taxes on
forest products, salt, and mines, as d) Neither 1 nor 2
well as imposing fines and marriage
taxes.

Which of the above given statements is/are Q.51) Consider the following statements
correct? about the changes, introduced in the
Mansabdari system by the Mughals, after
a) 1 and 2 only Akbar:
b) 2 and 3 only 1. Jahangir implemented a new
c) 1 and 3 only regulation called Du-Aspa Sih-Aspa
within the Sawar rank, specifically
d) 1, 2 and 3 targeting certain Mansabdars.
2. Shahjahan implemented the month-
scale in the Mansabdari system to
Q.50) Consider the following statements bridge the disparity between Jama
with respect to coinage during the reign of (estimated income) and hasil (actual
the Vijayanagara Empire: realisation).
3. Aurangzeb instituted the Mashrut
1. The coins feature various images rank and implemented the Khurak-i-
such as Hanuman, Garuda, Dawwab deduction, which aimed to
elephants, Shiva-Parvati, account for the costs associated
Lakshminarayana, Saraswati- with animal feed in the imperial
Brahma, and the royal emblem of stables.
Vijayanagara, Varaha.
How many of the statements given above
is/are correct?
.
IASBABA’S PRELIMS TEST SERIES (AIPTS) 2024 – TEST 5 (MEDIEVAL HISTORY)

a) Only one Q.54) With reference to Mughal Rule,


b) Only Two consider the following statements:
c) All Three
d) None 1. The Mughal court chronicles were
written in Urdu.
Q.52) With reference to the Medieval 2. The Ramayana was translated as the
history of India, the term ‘Zawabit’ Razmnama.
encompassed which of the following? 3. Persian became the language of
administration at all levels.
a) They were instruments used for land
How many of the statements given above
measurements for revenue
is/are correct?
assessment during the Mughal
period. 1. Only one
b) They were new techniques of 2. Only Two
irrigation developed during the Delhi 3. All Three
Sultanate. 4. None
c) These were the regulations framed
by Delhi Sultans to supplement the Q.55) Which of the following is/are the
Muslim Law. significant contribution(s) of Babur in India?
d) These were the regulations declared
by Allauddin Khalji to regulate the 1. He introduced gunpowder to India.
markets. 2. The Baburnama is the name of the
autobiography written by Babur
Q.53) In the Mughal empire, ‘Duaspa- himself.
Sihaspa’ means- 3. India could take a greater share in
great trans-Asian trade only after his
a) A Revenue system introduced by advent.
Akbar.
b) A decorative method introduced by How many of the statements given above
Shahjahan. is/are correct?
c) A sub rank in Sawar introduced by
a) Only one
Jahangir.
b) Only Two
d) A new painting method introduced
c) All Three
by Aurangzeb.
d) None
.
IASBABA’S PRELIMS TEST SERIES (AIPTS) 2024 – TEST 5 (MEDIEVAL HISTORY)

Q.56) With reference to Sher Shah’s Q.58) Consider the following pairs related
provincial administration, the "Shiqdar" was to the Mughal period
responsible for which of the following?
(Term) (Meaning)
a) Religious rites 1. Polaj Land which had
b) Collection of land revenue been fallow for
c) Law and order, and general two to three years
administration 2. Chachar Land which
d) Responsible for justice remained under
cultivation almost
Q.57) Which of the following statements is every year
correct about 'Khudkasht' during the 3. Nasaq It was a rough
Mughal period? calculation of the
amount payable
Statement - I: Khudkasht were the peasants
by the peas- ant
who owned the land which they tilled.
on the basis of
Statement - II: The Khudkasht often what he had been
belonged to a dominant caste, which paying in the past.
dominated the village society.
How many of the following pairs is/are
Which one of the following is correct in correctly matched?
respect of the above statements?
a) Only one pair
a) Both Statement-I and Statement-II b) Only two pairs
are correct and Statement-II is the c) Only three pairs
correct explanation for Statement-I. d) None of the pairs are correctly
b) Both Statement-I and Statement-II matched
are correct and Statement-II is not
the correct explanation for Q.59) Which of the following statements
Statement-I. are correct regarding Ahmad Shah I?
c) Statement-I is correct and but
1. He was considered as the real
Statement-II is incorrect.
founder of the Muzaffarid dynasty
d) Statement-I is incorrect but
of Malwa.
Statement-II is correct.
2. He shifted the capital from Patan to
the new city of Ahmedabad.
3. He drew on the rich architectural
traditions of the Jains of Gujarat to
.
IASBABA’S PRELIMS TEST SERIES (AIPTS) 2024 – TEST 5 (MEDIEVAL HISTORY)

devise a style of building which was Q.61) Consider the following statements
markedly different from Delhi. about the Battle of Haldighati:
4. He imposed Jizyah on the Hindus in
Gujarat, which had not been Statement – I: The Battle of Haldighati was
imposed on them earlier. fought due to conflict between Hindus and
Muslims on religious grounds.
How many of the statements given above
is/are correct? Statement – II: The Battle of Haldighati saw
Maharana Pratap and his forces engage in
1. Only one combat against the army of Mughal
2. Only Two Emperor Akbar.
3. Only Three
4. All four statements are correct Which one of the following is correct in
respect of the above statements?
Q.60) Consider the following statements
a) Both Statement-I and Statement-II
regarding Akbar’s reign:
are correct and Statement-II is the
1. Akbarnama deals with the laws and correct explanation for Statement-I.
the revenue system of Akbar’s b) Both Statement-I and Statement-II
empire. are correct and Statement-II is not
2. Religious teachers from foreign the correct explanation for
countries were also invited to Statement-I.
ibadatkhana before evolving Din-i- c) Statement-I is correct and but
Ilahi. Statement-II is incorrect.
3. The term Din-i-Ilahi signifies divine d) Statement-I is incorrect but
monotheism, representing the Statement-II is correct.
Religion of the one God.
Q.62) With reference to literature during
How many of the statements given above medieval period, which of the following
is/are correct? statements is/are correct?
a) Only one
b) Only Two 1. Persian was introduced in India by
c) All Three the Turks.
d) None 2. Firdausi was a poet of the Arabic
language.
3. Persian also became the language of
the upper classes apart from
administration and diplomacy.
.
IASBABA’S PRELIMS TEST SERIES (AIPTS) 2024 – TEST 5 (MEDIEVAL HISTORY)

4. Tuti Nama is a redaction of a c) Only Three


Sanskrit anthology titled d) All Four Statements are correct
Sukasaptati.
Q.65) Consider the following statements
How many of the statements given above
about Vallabhacharya:
is/are correct?
1. His philosophy is known as Shuddha
1. Only one
Advaita.
2. Only Two
2. He propounded the Pustimarga.
3. Only Three
3. He played a pivotal role in the
4. All Four Statements are correct
establishment of the Srinathji
Temple in Nathdwara, Rajasthan.
Q.63) The systems of ‘Watans’ and
‘Saranjami’ are associated with which How many of the statements given above
among the following kingdoms? is/are correct?

a) Mughals 1. Only one


b) Rajputs 2. Only Two
c) Marathas 3. All Three
d) Sikhs 4. None

Q.64) Consider the following statements Q.66) Consider the following pairs:
about Sufism in India:
(Saints) (State)
1. Moinuddin Chishti was the first Sufi 1. DaduDayal : Madhya
saint to come to India and settle. Pradesh
2. Ziyarat refers to pilgrimage to the 2. Ravi Das : Punjab
tombs of Sufi saints. 3. BhagatPipa : Bihar
3. Maktubat is compilations of
How many of the following pairs is/are
conversations of the Sufi saints.
correctly matched?
4. Malfuzat is collections of letters
written by the Sufi masters, a) Only one pair
addressed to their disciples and b) Only two pairs
associates. c) Only three pairs
d) None of the pairs are correctly
How many of the statements given above
matched
is/are correct?

a) Only one
b) Only Two
.
IASBABA’S PRELIMS TEST SERIES (AIPTS) 2024 – TEST 5 (MEDIEVAL HISTORY)

Q.67) Consider the following statements Q.69) Which of the following is/are the
about Ramanujacharya: social impacts of Bhakti movement on
Society?
1. His philosophy is called
Visishtadvaita. 1. Improvement in the social relation
2. He challenged the monist ideology between the Hindus and Muslims.
of AdiSankara. 2. The better social status of lower
3. He influenced some temple castes.
authorities to permit the social 3. Enrichment of literature.
groups outside the Varnashram 4. Development of composite art in the
system to enter into the temple at society.
least once a year.
How many of the statements given above
Which of the statements given above is/are is/are correct?
correct?
1. Only one
a) 1 and 2 only 2. Only Two
b) 1 and 3 only 3. Only Three
c) 2 and 3 only 4. All Four Statements are correct
d) 1, 2 and 3
Q.70) Consider the following statements
Q.68) Given the following clues, information about ancient Indian philosophies:
about the personality is presented:
1. Statement – I: According to both
1. Many of his composition are found Samkhya and Nyaya School, a
in Adi-granth sahib. person can attain salvation through
2. His poetry contained Islamic, Vedic the acquisition of knowledge.
as well as yogic traditions. 2. Statement – II: Vaisheshika School,
3. He was the follower of Bhakti saint which propounded the atom theory,
Ramananda. considered debates on heaven and
salvation as meaningless.
Which of the following personality has been
discussed above? Which one of the following is correct in
respect of the above statements?
a) Kabir
b) Guru nanak a) Both Statement-I and Statement-II
c) Guru Gobind Singh are correct and Statement-II is the
d) Tulsidas correct explanation for Statement-I.
b) Both Statement-I and Statement-II
are correct and Statement-II is not
.
IASBABA’S PRELIMS TEST SERIES (AIPTS) 2024 – TEST 5 (MEDIEVAL HISTORY)

the correct explanation for 2. Only Two


Statement-I. 3. All Three
c) Statement-I is correct and but 4. None
Statement-II is incorrect.
d) Statement-I is incorrect but Q.73) Consider the following pairs:
Statement-II is correct.
Title of the book) (Author)
Q.71) Consider the following statements 1. Humayunnama Abdul Hamid
about agrarian relations during the Mughal Lahori
Empire: 2. Badshahnama Gulbadan
Begum
1. Muzarian denotes the peasant class. 3. Alamgirnama Muhammad
2. Pahi-Kashta represented the Kazim
inhabitants of the village who
owned and cultivated the lands. How many of the following pairs is/are
3. Peasant lands were non-transferable correctly matched?
and could not be bought or sold.
a) Only one pair
How many of the statements given above b) Only two pairs
is/are correct? c) Only three pairs
d) None of the pairs are correctly
a) Only one
matched
b) Only Two
c) All Three
Q.74) Consider the following statements
d) None
about ‘Mahzarnama’:

Q.72) Consider the following statements 1. It was a declaration issued by


about agriculture during the Mughal period: Aurangzeb and was also called as
the ‘Decree of Infallibility’.
1. Jins-i Kamil refers to the crops that
2. It was drafted by Sheik Mubarak and
were cash crops in nature.
AbulFazl.
2. Jahangir issued a prohibition on the
3. Under it, the emperor became the
cultivation of tobacco.
final interpreter of Quran.
3. Bengal was renowned for its
significant production of sugar. Which of the statements given above is/are
correct?
How many of the statements given above
is/are correct? a) 1 and 2 only
1. Only one b) 1 and 3 only
c) 2 and 3 only
.
IASBABA’S PRELIMS TEST SERIES (AIPTS) 2024 – TEST 5 (MEDIEVAL HISTORY)

d) 1, 2 and 3 Q.77) With reference to the capital city of


the Vijayanagara Empire (Hampi), consider
Q.75) Consider the following statements the following statements:
about the Dahsala system:
1. It is famous as a pilgrimage center
1. It was introduced by Raja Todar Mal. for both the Hindu and Jain
2. It was a system of annual religions.
assessment. 2. It is situated on the banks of the
3. It was extended to all parts of the Krishna River.
Mughal administration by Akbar. 3. The Group of Monuments at Hampi
was added to the UNESCO World
How many of the statements given above
Heritage Sites in 1986.
is/are correct?
How many of the statements given above
1. Only one
is/are correct?
2. Only Two
3. All Three a) Only one
4. None b) Only Two
c) All Three
Q.76) “No one in the whole world has ever d) None
seen or heard of a city like Vijayanagara.
Vijayanagara is surrounded by a seven Q.78) Consider the following statements
round baladya fort. The city is dotted with with respect ‘Economic Life’ during the
many markets, parks, wide streets and Vijayanagara Empire:
buildings. Rose and jasmine flowers were
loved by the people of Vijayanagara. Gems, 1. The primary occupation of the
diamonds and turquoise were sold on the people in the Vijayanagar Empire
streets without any fear.” was trade and commerce.
Which of the following foreign traveler is 2. The primary exports were cotton
using the above words to describe the and silk clothing, spices, rice, iron,
Vijayanagara Empire? saltpetre, and sugar.
3. The main imports were horses,
a) Domingo Paes pearls, copper, coral, mercury, China
b) Barbosa silk, and velvet clothing.
c) Nicolo-di-khanti
Which of the above given statements is/are
d) Abdur Razzak
correct?

a) 1 and 2 only
b) 2 and 3 only
.
IASBABA’S PRELIMS TEST SERIES (AIPTS) 2024 – TEST 5 (MEDIEVAL HISTORY)

c) 1 and 3 only b) Alauddin Bahman Shah, Firoz Shah


d) 2 only Bahmani, Mahmud Gawan, Ahmad
Shah
Q.79) With reference to conflicts between c) Alauddin Bahman Shah, Ahmad
Vijayanagara and Bahmani Kingdoms, Shah, Firoz Shah Bahmani, Mahmud
consider the following statements: Gawan
d) Alauddin Bahman Shah, Firoz Shah
Statement – I: The conflicts between the Bahmani, Ahmad Shah, Mahmud
Vijayanagara Empire and the Bahmani Gawan
Sultanate can be simplified as a mere clash
between Hindus and Muslims. Q.81) “He was an advocate of education
Statement – II: Another reason for the and literature, established 'Madrasa' in
conflicts was the Raichur doab, an area of Bidar, where he completed the exceptional
great significance as it was fertile and madrasa building in 1472 AD and began
abundant in mineral resources. delivering lectures on various subjects,
including philosophy, geography,
Which one of the following is correct in mathematics, and language. Furthermore,
respect of the above statements? he generously offered free lodging and
board to the college's students and faculty
a) Both Statement-I and Statement-II
members. The Madrasa library housed
are correct and Statement-II is the
approximately 3,000 books, and he is a
correct explanation for Statement-I.
scholar himself, made significant
b) Both Statement-I and Statement-II
contributions to the advancement of
are correct and Statement-II is not
education”.
the correct explanation for
Statement-I. Identify which personality is described in
c) Statement-I is correct and but the above paragraph?
Statement-II is incorrect.
d) Statement-I is incorrect but a) Ibrahim Adilsha I
Statement-II is correct. b) Muhammad Shah I
c) Muhammad Gawan
Q.80) What is the chronological order of the d) Ibrahim Adilsha II
following kings of Bahmani Sultans dynasty?
Q.82) Consider the following pairs with
a) Ahmad Shah, AlauddinBahman respect to council of ministers in Bahmani
Shah, Firoz Shah Bahmani, Mahmud Sultans Dynasty:
Gawan
(NAME) (MINISTRY)
Amir-i-jumla Minister of finance
.
IASBABA’S PRELIMS TEST SERIES (AIPTS) 2024 – TEST 5 (MEDIEVAL HISTORY)

Sadr-i-jahan Minister of foreign Q.84) Match the foreign travelers with their
affairs corresponding countries:
Wasir-i-ashraf Minister of religious
affairs Foreign travelers Countries
1. Nicolo Conti A. Russia
How many of the following pairs is/are 2. Athanasius Nikitin B. Italy
incorrectly matched? 3. Abdur Razzaq C. Portugal
4. Domingo Paes D. Persia
a) Only one pair
b) Only two pairs
Select the answer by using the code below:
c) Only three pairs
d) None of the pairs are correctly a) 1-B, 2-A, 3-C, 4-D
matched b) 1-D, 2-C, 3-B, 4-A
c) 1-A, 2-B, 3-D, 4-C
Q.83) Match the authors with their d) 1-B, 2-A, 3-D, 4-C
corresponding literary works:
Q.85) With reference to the taxation system
Authors Literary Works of Krishna Deva, the ruler of Vijayanagara,
Nandithimmana Sri Kalahastisqara consider the following statements:
Shatakam
Statement – I: The tax rate on land was
Tenali Ramakrishna Rajasekhara
fixed depending on the quality of the land.
Charitam
Doorzati Vanivilasam Statement – II: Private owners of workshops
Madayagari Mallanna Udbhataradhya paid an industry's tax and the tax rate was
Charitam calculated based on the nature and size of
Pingli Suranna Raghava the industry.
Pandaveiyamu
Which one of the following is correct in
respect of the above statements?
Select the answer by using the code below:
1. Both Statement-I and Statement-II
a) 1-C, 2-D, 3-E, 4-B, 5-A are correct and Statement-II is the
b) 1-D, 2-C, 3-A, 4-B, 5-E correct explanation for Statement-I.
c) 1-C, 2-D, 3-A, 4-B, 5-E 2. Both Statement-I and Statement-II
d) 1-C, 2-D, 3-A, 4-E, 5-B are correct and Statement-II is not
the correct explanation for
Statement-I.
3. Statement-I is correct and but
Statement-II is incorrect.
.
IASBABA’S PRELIMS TEST SERIES (AIPTS) 2024 – TEST 5 (MEDIEVAL HISTORY)

4. Statement-I is incorrect but Q.88) With reference to the Sufi Movement,


Statement-II is correct. consider the following statements:

Q.86) With reference to the Sufism, 1. In the 13th century, the Sufis were
consider the following statements: divided into 12 orders, also known
as Silsilahs.
1. Sufism does not emphasize the inner 2. Amrit-Kund had been translated
search for God and a direct personal from Sanskrit into Persian.
experience of the divine as a 3. The Sufi system was dependent on
mystical and spiritual Islamic belief the bond between students and
system. their Pir.
2. Sufism has had a significant
How many of the statements given above
influence on Islamic culture,
is/are correct?
particularly in the fields of art,
literature, and music. 1. Only one
3. Sufis express their spiritual 2. Only Two
experiences through poetry, music, 3. All Three
and dance. 4. None
How many of the statements given above
Q.89) Which of the following foreign
is/are correct?
travelers who visited the Vijayanagara
a) Only one Empire during the reign of Krishna Deva
b) Only Two Raya?
c) All Three
d) None 1. Domingo Paes
2. Duarte Barbosa
Q.87) Which of the following Sufi saints is 3. Nicolo-de-conti
known as ‘Mehbub-e-llahi’? 4. Fernao Nuniz

Select the correct answer by using the code


a) Baba Farid
below:
b) Shaik Nasir-ud-din chirag-i-Dehalvi
c) ShaikNizam-ud-din Auliya a) 1, 2 and 3 only
d) ShaikMuin-ud-din chisti b) 1 and 2 only
c) 2, 3 and 4 only
d) 1, 2, 3 and 4
.
IASBABA’S PRELIMS TEST SERIES (AIPTS) 2024 – TEST 5 (MEDIEVAL HISTORY)

Q.90) Match the terms of Sufism with its arches, and it is said to be one of the
related meaning: largest of its kind in the world.
2. The central hall of Gol Gumbaz
Terms Meaning contains the tombs of Muhammad
1. Wali A. Pilgrimage Adil Shah, his wives, and his
2. Khalifa B. A chain daughters.
3. Silsila C. Successor 3. It has been declared a protected
4. Ziyarat D. Friend of God monument by the Archaeological
Survey of India.
Select the answer by using the code below:
How many of the statements given above
a) 1-B, 2-A, 3-C, 4-D is/are correct?
b) 1-D, 2-C, 3-B, 4-A
a) Only one
c) 1-A, 2-B, 3-D, 4-C
b) Only Two
d) 1-B, 2-A, 3-D, 4-C
c) All Three
d) None
Q.91) Consider the following statements
with respect to Ibrahim Adil Shah II:
Q.93) Who among the following Mughal
1. He made efforts to foster cultural Emperors initiated the 'Phool Walon Ki Sair'
harmony between various religious festival as a means of fostering Hindu-
communities, including Shia and Muslim unity?
Sunni Muslims as well as Hindus.
a) Shahajahan
2. He abolished music in his court.
b) Akbar II
3. He had written a book titled Kitab-E-
c) Jahangir
Navras in Dakhani language.
d) Jahandar Shah
Which of the above given statements is/are
correct? Q.94) What is the chronological order of the
following kings of Mughal Empire?
a) 1 and 3 only
b) 2 and 3 only a) Jahandar Shah, Farrukhsiyar,
c) 1 and 2 only Muhammad Shah Rangeela, Shah
d) 1, 2 and 3 Alam II
b) Jahandar Shah, Muhammad Shah
Q.92) With reference to the Gol Gumbaz, Rangeela, Farrukhsiyar, Shah Alam II
consider the following statements: c) Farrukhsiyar, Jahandar Shah,
Muhammad Shah Rangeela, Shah
1. The dome of Gol Gumbaz is Alam II
supported by eight intersecting
.
IASBABA’S PRELIMS TEST SERIES (AIPTS) 2024 – TEST 5 (MEDIEVAL HISTORY)

d) Jahandar Shah, Farrukhsiyar, Shah Q.97) Who was the infamous Qazi Fazihat
Alam II, Muhammad Shah Rangeela or Qazi Fazilat during the reign of Sher
Shah?
Q.95) With reference to the Maratha
Empire, consider the following statements: a) Governor of Bengal
b) Governor of Delhi
1. Raigad was the capital of the c) Commander in chief
Maratha Empire. d) Revenue accountant
2. Maratha were experts in Guerilla
warfare. Q.98) Who were the sons of Shahjahan
3. Shivaji adorned himself with the title involved in the war of succession that took
of Chatrapati in 1674. place in 1657-58 A.D.?

Which of the above given statements is/are a) KamBaksh, Murad, DaraShikoh,


correct? Aurangzeb
b) Aurangzeb, DaraShikoh, Shuja,
a) 1 and 3 only
KamBaksh
b) 2 and 3 only
c) DaraShikoh, Aurangzeb, Murad,
c) 1 and 2 only
Shuja
d) 1, 2 and 3
d) DaraShikoh, Kamran, Aurangzeb,
Murad
Q.96) With reference to Chola rulers and
their titles consider the following pairs:
Q.99) Consider the following statements:
Chola ruler Titles
Statement – I: The Bhakti movement was
1. Rajendra Chola Pandita Chola contemporary to the Sufi movement.
2. Rajaraja I Ponniyan Selvan
3. Rajadhiraja Jayamkonda Chola Statement – II: The Bhakti movement
stressed on the unity of God.
How many of the following pairs is/are
Which one of the following is correct in
correctly matched?
respect of the above statements?
a) Only one pair
a) Both Statement-I and Statement-II
b) Only two pairs
are correct and Statement-II is the
c) Only three pairs
correct explanation for Statement-I.
d) None of the pairs are correctly
b) Both Statement-I and Statement-II
matched
are correct and Statement-II is not
the correct explanation for
Statement-I.
.
IASBABA’S PRELIMS TEST SERIES (AIPTS) 2024 – TEST 5 (MEDIEVAL HISTORY)

c) Statement-I is correct and but


Statement-II is incorrect.
d) Statement-I is incorrect but
Statement-II is correct.

Q.100) With respect to Sher Shah Suri,


Consider the following statements:

1. Sher Shah Suri is also referred to as


Farid Khan.
2. He is credited with constructing the
Grand Trunk Road in India.
3. Sher Shah was assassinated by
Humayun.

Select the correct answer by using the


codes below:

a) 1 only
b) 1 and 2 only
c) 3 only
d) 1, 2 and 3
.
IASBABA’S PRELIMS TEST SERIES (AIPTS) 2024 – TEST 5 (MEDIEVAL HISTORY)

Q.1) Solution (b)

Explanation:

• The Tripartite Struggle, also known as the Kannauj Triangle Wars, took place in the 8th
and 9th centuries, between the three great Indian dynasties, the Palas, the Pratiharas,
and the Rashtrakutas, for control over the Kannauj area of northern India.

• The Palas ruled India's eastern regions (Bengal region), while the Pratiharas oversaw
India's north-western regions (Avanti-Jalaor region) and the Deccan area of India was
dominated by the Rastrakutas. This war lasted for two centuries and was finally won by
the Rajput Pratihara emperor Nagabhata II, who established the city as the capital of
the Pratihara state, which ruled for nearly three centuries. (Hence statement 1 is
correct)

• During the early medieval period, Kannauj was seen as a sign of status and authority.
Kannauj was the former capital of Harshvardhana's empire and control of it represented
the political domination over northern India. Control of Kannauj also meant control of
the Central Gangetic valley, which was rich in resources and hence strategically and
commercially significant. Because it was connected to the silk road, this location was
ideal for trade and commerce. The desire to plunder through warfare attracted the
Rashtrakutas to Kannauj. (Hence statement 2 is correct)

• The prolonged tripartite struggle for Kannauj persisted for nearly two centuries until it
ultimately resulted in the triumph of Nagabhata II, the ruler of the Gurjara-Pratihara
dynasty. Under his reign, Kannauj became the capital of the Gurjara-Pratihara empire,
which continued to govern for nearly three centuries. (Hence statement 3 is incorrect)

Important Points/Value Add:

• After Harsha, Kannauj garnered significant attention from various forces, attaining a
position akin to Magadh during the Gupta era.

• Both Harsha and Yashovarman contributed to establishing it as a symbol of imperial


power. Securing control over Kannauj became crucial for aspiring rulers aiming to
become Chakravartis in northern India.

• Additionally, Kannauj's economic significance has witnessed substantial growth, further


enhancing its allure, not just politically but also from an economic standpoint.

• The Pratiharas, who derived their name from the word pratihara (Sanskrit:
“doorkeeper”) are seen as a tribal group or a clan of the Gurjaras.
.
IASBABA’S PRELIMS TEST SERIES (AIPTS) 2024 – TEST 5 (MEDIEVAL HISTORY)

Q.2) Solution (d)

Explanation:

The Gurjara Pratihara Kingdom was a powerful Indian dynasty that ruled over parts of North
India from the 8th to the 11th centuries CE. Some of the important rulers of the Gurjara
Pratihara Kingdom are:

• Nagabhata I (730-760 CE): He was the founder of the Gurjara Pratihara dynasty and
expanded his kingdom to include parts of present-day Rajasthan, Haryana, and Uttar
Pradesh.

• Vatsaraja (780–800): or Vatsraja was an Emperor of Northern India's Pratihara dynasty.


He was Nagabhata I's grand-nephew, and his mother was Queen Bhuyikadevi. He was
the first ruler of Rajasthan to triumph over the far-flung regions of Kanauj and Bengal.

• Bhoja I (836-885 CE): He is considered one of the most important rulers of the Gurjara
Pratihara dynasty. During his reign, the kingdom reached its peak of power and
influence.

• Mahendrapala I (885-910 CE): He was a powerful ruler who continued the expansion of
the Gurjara Pratihara Kingdom and defeated several rival kingdoms.

• Mihira Bhoja (892-910 CE): He was the son of Mahendrapala I and ruled over the
kingdom during a period of great cultural and artistic flourishing.

• Mahipala I (912-944 CE): He was another important ruler who extended the kingdom's
boundaries and patronized art and culture.

• Bhoja II (1010-1055 CE): He was the last great ruler of the Gurjara Pratihara dynasty and
is known for his patronage of literature and the arts.

Q.3) Solution (a)

Explanation:

• Rajasekhara was a great poet and dramatist of his age. He lived in the court of the
Pratiharas.
.
IASBABA’S PRELIMS TEST SERIES (AIPTS) 2024 – TEST 5 (MEDIEVAL HISTORY)

• The Karpuramanjari was his first play. He was regarded as the teacher of Nirbhaya or
Mahendrapala of Kannauj.

• The Balardmdyana was produced at his request. But again, the unfinished Balabharata
was written for the Pratihara ruler Mahipala I, whose rule begins probably in 914 A.D.

• His contributions to Sanskrit literature have been highly regarded and have influenced
many subsequent writers and poets.

These are the some of the books attributed to poet Rajshekhara:

• Karpuramanjari: A famous play written in Sauraseni Prakrit to please his wife,


avantisundari, a woman of taste and accomplishment. He is perhaps the only ancient
Indian poet to acknowledge a woman for her contributions to his literary.

• Kavya Mimamsa: A practical guide for poets that explains the elements and
composition of a good poet.

• Viddhasalabhanjika: It was written for Yuvaraja Keyuravarsa of Tripuri

• Balabharata: It is based on the Mahabharata that depicts the marriage of Draupadi, the
Pandavas' loss of kingdom, the gambling scene with the ill-usages of Draupadi and the
Pandavas' departure to the forest.

• Bala Ramayana: Though the drama is based on the Ramayana but the love of Ravana
has been made the dominating factor.

Q.4) Solution (b)

Explanation:

• According to the Khalimpur copper plate inscription of Dharmapala, in order to rescue


the people from Matsya-Nyaya (a period of anarchy), Gopala founded Pala dynasty
when he was elected the king by notable men of the realm. (Hence statement 1 is
correct).

• Buddhism and Hinduism were the dominant religions throughout the Pala Dynasty. They
were largely Buddhists and practiced Mahayana Buddhism. After King Harsha
Vardhana's reign, Buddhism was on the verge of extinction. However, the arrival of the
Palas reignited interest in Buddhism across the Indian subcontinent. (Hence statement 2
is correct)
.
IASBABA’S PRELIMS TEST SERIES (AIPTS) 2024 – TEST 5 (MEDIEVAL HISTORY)

• Shaivism and Vaishnavism were also patronized by them. The King made land
concessions to brahmanas and priests. Palas built great monasteries throughout their
reign. One of the major viharas built by the Pala emperors is Somapura Mahavihara
(now in Bangladesh). The spread of Mahayana Buddhism occurred during the Palas in
nations such as Tibet, Nepal, Bhutan, Myanmar, and Indonesia.

• The Nalanda University was not founded by the Pala rulers, but it flourished under their
patronage. The university was founded during the Gupta period in the 5th century CE
and reached its peak during the reign of the Pala rulers in the 8th to 12th centuries CE.
(Hence statement 3 is incorrect).

Important Points/Value Add:

• The Nalanda University, which had been famous all over the eastern world, was revived
by Dharmapala and 200 villages were set apart for meeting its expenses. He also
founded the Vikramasila University at Bhaglapur (Bihar), which became second only to
the Nalanda University in fame.

• About Nalanda University: It was a renowned ancient center of learning in India.


Located in present-day Bihar, it flourished from the 5th century to the 12th century CE.
It attracted scholars from different parts of the world and was considered one of the
most prestigious educational institutions of its time. Nalanda University was known for
its vast library, which housed a vast collection of manuscripts and texts covering various
disciplines such as philosophy, astronomy, mathematics, medicine, and literature. The
university's curriculum included subjects like Buddhist philosophy, logic, and Sanskrit
grammar. Nalanda University's decline and eventual destruction occurred during the
invasions of Muslim armies in the 12th century, marking the end of an important
chapter in the history of ancient education and scholarship in India.

Q.5) Solution (b)

Explanation:

• The Rashtrakuta rulers were known for their religious tolerance. They were patrons of
both Hinduism and Jainism, and even built temples dedicated to both religions. They
also supported Buddhism, and some of their inscriptions mention the construction of
Buddhist shrines. (Hence statement 1 is correct).
.
IASBABA’S PRELIMS TEST SERIES (AIPTS) 2024 – TEST 5 (MEDIEVAL HISTORY)

• The famous rock-cut temple of Siva (Kailasa Temple) at Ellora was built by one of the
Rashtrakuta kings, Krishna I, in the 9th century. His successor, Amoghavarsha, is said to
have been a Jain, but he also patronized other faiths. (Hence statement 2 is incorrect).

• The Rashtrakutas allowed the Muslim traders to settle and permitted Islam to be
preached in their dominions. This tolerant policy helped to promote foreign trade,
which enriched the Rashtrakutas.

• An important feature of that period was the rise in the Deccan of the hereditary
revenue officers, called nad gavundas or desa gramakutas. They appear to have
discharged the same functions as the deshmukhs and deshpandes of the later times in
Maharashtra. (Hence statement 3 is correct).

Important Points/Value Add:

• Kailasa Temple- The iconic rock-cut temple of Siva at Ellora (Aurangabad district of
Maharashtra)was constructed by Krishna I, a prominent king of the Rashtrakuta dynasty.

• This magnificent temple, carved out of a single rock, showcases the remarkable
architectural and artistic skills of the Rashtrakuta period.

• Dedicated to Lord Shiva, the temple at Ellora stands as a testament to the dynasty's
patronage of religion and art.

• Its intricate carvings, elaborate sculptures, and detailed architectural features continue
to inspire awe and admiration, attracting visitors from around the world.

• The rock-cut temple of Siva at Ellora remains a remarkable symbol of the rich cultural
heritage of ancient India.
.
IASBABA’S PRELIMS TEST SERIES (AIPTS) 2024 – TEST 5 (MEDIEVAL HISTORY)

Q.6) Solution (b)

Explanation:

• Dantidurga (also known as Dantivarman), a feudatory of the Badami Chalukyas,


defeated their King Kirtivarman II, who later founded the Rashtrakuta kingdom by taking
control of the northern regions of the Chalukyan Empire and thus ascended to the
throne in 733 CE. (Hence statement 1 is correct).

• Amoghavarsha: He was a patron of literature and was an accomplished scholar in


Kannada and Sanskrit himself. He wrote Kavirajamarga – the earliest Kannada work on
Poetics and the Prashnottara Ratnamalika in Sanskrit, which is considered as a writing
of high merit and was later, translated into Tibetan Langauage. (Hence statement 2 is
correct).

• The Arab traveler Sulaiman described Amoghavarsha as one of the "four great kings of
the world along with the Caliph of Bagdad, the King of Constantinople, and the Emperor
of China, because for his religious temperament, his interest in the arts and literature
and his peace-loving nature.

• Historian Panchamukhi has compared him to emperor Ashoka and called ‘Ashoka of the
South’, and also compared to Gupta King Vikramaditya in giving patronage to men of
letters. (Hence statement 3 is incorrect).

Q.7) Solution (c)

Explanation:

• Jinasena was a renowned Jain scholar and philosopher who lived in the 9th century CE,
during the reign of the Rashtrakuta king Amoghavarsha I.

• He was a follower of the Digambara sect of Jainism and is best known for his work called
"Adipurana" and “Mahapurana”.

• Rastrakuta king Amoghavarsha was his disciple.

• Jinasena is also believed to have written a commentary on the "Tattvartha Sutra", which
is a Jain philosophical text that explains the nature of reality and the path to liberation.
.
IASBABA’S PRELIMS TEST SERIES (AIPTS) 2024 – TEST 5 (MEDIEVAL HISTORY)

His works have had a profound impact on the development of Jainism and are still
studied and revered by Jains around the world.

Q.8) Solution (b)

Explanation:

• The most important cause for the failure of Rajput’s was that they lacked unity. They
were divided by factions.

• The Rajput princes exhausted one another by their mutual conflicts. On the other hand,
Muslim invaders were united by their religious affiliations as well as effective leadership.
Their religious zeal and their greed for the greater wealth of India provided stimulus to
them. (Hence statement 1 is correct).

• Many Hindu states were declining in power. Their military methods were out of date
and far inferior to those of invaders. Indians continued to rely on elephants while the
Muslims possessed quick moving cavalry. (Hence statement 2 is incorrect).

• Among the Hindus, the duty of fighting was confined to a particular class, the
Kshatriyas. (Hence statement 3 is incorrect).

• Moreover, the Hindus were always on the defensive side, which was always a weak
position. (Hence statement 4 is correct).

Important Points/Value Add:

• Economic Factors: Economic instability and decline played a crucial role in the downfall
of native kingdoms. Trade routes were disrupted, which led to a decline in commerce
and revenue. Additionally, mismanagement of resources, excessive taxation, and a lack
of economic reforms contributed to economic hardships, further weakening the
kingdoms.

• Feudal System: The feudal system prevalent in many native kingdoms led to a
fragmentation of power. Feudal lords held significant authority, often challenging the
central authority of the kingdom and undermining its stability and governance.

• Social and Cultural Changes: Social and cultural transformations, such as the rise of
regional identities and the influence of new religious and cultural movements, also
impacted the stability of native kingdoms. The emergence of new religious sects and the
.
IASBABA’S PRELIMS TEST SERIES (AIPTS) 2024 – TEST 5 (MEDIEVAL HISTORY)

decline of patronage towards established religions caused societal divisions and


conflicts, further destabilizing the kingdoms.

• Technological Advancements: The native kingdoms struggled to keep up with the


technological advancements of invading forces. Superior military technology, such as
the use of cavalry and advanced weaponry, provided an edge to the foreign invaders,
making it difficult for the native kingdoms to defend themselves effectively.

• Lack of Centralized Administration: Many native kingdoms lacked efficient centralized


administrative structures and bureaucratic systems. This led to issues in governance, law
enforcement, and public administration, creating a sense of instability and
dissatisfaction among the populace

Q.9) Solution (b)

Explanation:

• The king was the most important person in the Chola administration. All authority
rested in his hands, although he did rely on the advice and counsel of his ministers.
(Hence statement 1 is incorrect).

• The basic unit of administration was the Nadu, which consisted of a number of villages
having close kinship ties and other close associations. The number of Nadus increased as
fresh lands were brought under cultivation by means of irrigation works, such as ponds,
wells, etc., and by converting hill or tribal people into agriculturists. (Hence statement 2
is correct).

• As the governors of the provinces were not selected from the local people through
elections. Princes of the royal family were appointed the Governors of the provinces.
(Hence statement 3 is incorrect).

• The gathering of the adult men in the Brahman villages, which were called the
Agraharas. These were villages with Brahman settlements in which most of the land
were rent-free. These villages enjoyed a large measure of autonomy. The affairs of the
village were managed by an executive committee, to which educated persons owning
property were elected either by drawing lottery or by rotation. (Hence statement 4 is
correct).
.
IASBABA’S PRELIMS TEST SERIES (AIPTS) 2024 – TEST 5 (MEDIEVAL HISTORY)

Q.10) Solution (b)

Explanation:

• The Uttaramerur inscription (Kanchipuram, Tamil Nadu), dated around 920 A.D. in the
reign of Parantaka Chola [907-955 A.D.], testifies to the historical fact that nearly 1,100
years ago, a village had an elaborate and highly refined electoral system and even a
written constitution prescribing the mode of elections.

• “The inscription found there is like a local constitution for the gram sabha. It tells how
the assembly should be run, what should be the qualification of members, what should
be the process to elect the members, and how a member would be disqualified.”

• The inscription gives details of the functioning of the local sabha, i.e. the village
assembly. A sabha was an assembly exclusively of Brahmans and had specialized
committees tasked with different things.

• The Uttaramerur inscription details how members were selected, the required
qualifications, their roles and responsibilities, and even the circumstances in which they
could be removed.

https://indianexpress.com/article/explained/explained-history/uttaramerur-inscription-pm-
modi-8556704/

Q.11) Solution (c)

Explanation:

• During the Rajendra Chola expedition to the Gangetic Valley in the 11th century CE, the
ruler of the Pala dynasty was likely to be Mahipala I (Hence option (c) is correct)

• Rajendra's armies embarked on an extraordinary expedition after capturing both the


Western and Eastern Chalukya fronts.

• Rajendra's forces continued to march through Kalinga to the Ganges around 1019 C.E.

• The Emperor himself advanced up to the Godavari River to protect the expeditionary
force's rear.

• The Chola army eventually made its way to the Pala kingdom of Bengal, where they met
and defeated Mahipala.
.
IASBABA’S PRELIMS TEST SERIES (AIPTS) 2024 – TEST 5 (MEDIEVAL HISTORY)

• According to the Tiruvalangadu Plates, the campaign lasted less than two years, during
which many northern kingdoms felt the Chola army's might.

• According to the inscriptions, Rajendra defeated the armies of Ranasura and entered
the land of Dharmapala and subdued him, and thus he reached the Ganges and caused
the water river to be brought back to the Chola country by the conquered kings.

Q.12) Solution (c)

Explanation:

• The early Chola temples were found at Narthamalai and Kodumbalur in Pudukottai
district, and at Srinivasanallur in Tiruchirappalli district. On the other hand, the
Kampaharesvara temple at Tribhuvanam and the Airavathesvara temple at Darasuram
in Tanjore district are examples of the later Chola temples. (Hence statement 1 is
incorrect).

• The inscriptions at Ennayiram, Thirumukkudal and Thirubhuvanai provide details of the


colleges which existed in the Chola kingdom. Education was given much importance
under the Cholas. Besides the temples and mathas as educational centres, several
educational institutions also flourished. (Hence statement 2 is correct).

• Commercial contacts between the Chola Empire and China, Sumatra, Java and Arabia
were extensively prevalent. Arabian horses were imported in large numbers to
strengthen the cavalry. (Hence statement 3 is correct).

Important note about Chola dynasty

The Chola dynasty was a prominent and influential dynasty that ruled over a major part of
South India from the 9th to the 13th centuries. Here are some key points about the Chola
dynasty:

• Origin and Capital: The Chola dynasty originated in the region of Tamil Nadu, with its
capital initially at Uraiyur and later shifted to Thanjavur (also known as Tanjore).

• Political Power: The Cholas emerged as a powerful political force under the leadership
of King Vijayalaya Chola, who established the dynasty in the 9th century. They went on
to become one of the longest-ruling dynasties in the history of southern India.

• Maritime Trade and Naval Power: The Cholas excelled in maritime trade and
established a vast empire that extended across South India, Sri Lanka, parts of Southeast
.
IASBABA’S PRELIMS TEST SERIES (AIPTS) 2024 – TEST 5 (MEDIEVAL HISTORY)

Asia, and the Maldives. They developed a formidable navy, which enabled them to exert
their influence over trade routes and maintain control over overseas territories.

• Administrative and Cultural Achievements: The Cholas were known for their efficient
administrative system, which included a well-organized bureaucracy and a network of
local governance. They also made significant contributions to art, literature, and
architecture. The magnificent Brihadeeswarar Temple in Thanjavur, built by Rajaraja
Chola I, is a UNESCO World Heritage Site and a testament to the architectural brilliance
of the dynasty.

• Temple Building: The Chola rulers were avid patrons of temple construction and played
a crucial role in the development of Dravidian architecture. Their temples, characterized
by intricate carvings, towering gopurams (gateways), and grand sculptures, remain
iconic symbols of South Indian temple architecture.

Q.13) Solution (c)

Explanation:

• The land revenue department was well organized. It was called as puravuvari
thinaikalam. All lands were carefully surveyed and classified for assessment of revenue.
The residential portion of the village was called urn nattam. These residential lands and
other lands, such as the lands belonging to the temples, were exempted from tax.
(Hence statement 1 is correct).

• Under the Cholas, the village was divided into 30 wards and each was to nominate its
members to the village council. The qualifications to become a ward member were:

• Ownership of at least one fourth veli of land.

• Own residence.

• Above thirty years and below seventy years of age.

• Knowledge of Vedas. (Hence statement 2 is correct).

• The Cholas maintained a regular standing army consisting of elephants, cavalry, infantry
and navy. They paid special attention to their navy. The naval achievements of the
Tamils reached its climax under the Cholas. They controlled the Malabar and
Coromandal coasts. In fact, the Bay of Bengal became a Chola Lake for some time.
(Hence statement 3 is correct).
.
IASBABA’S PRELIMS TEST SERIES (AIPTS) 2024 – TEST 5 (MEDIEVAL HISTORY)

Q.14) Solution (d)

Explanation:

• Rajendra I was a ruler of the Chola Empire and the conqueror. The Pandya, Kerala and
Mysore regions and also Sri Lanka formed part of the empire. He was the son of the
famous Chola king, Rajaraja I, and is considered one of the greatest rulers of the Chola
dynasty. Rajendra I is known for his military conquests and his extensive empire-
building, which extended the Chola Empire's boundaries to include Sri Lanka, parts of
Southeast Asia, and the Maldives.

• He also built the Gangaikonda Cholapuram temple and established the Chola kingdom
as a major center of culture and learning. He also excavated a large irrigation tank,
called Cholagangam, on the western side of the city.

• He defeated Jayasimha II, the Western Chalukyan king and as a result, the river
Tungabadhra was recognised as his northern boundary and a boundary between the
Cholas and Chalukyas. Another famous venture of Rajendra was his naval expedition to
Kadaram or Sri Vijaya, which was a complete success. He assumed the title
Kadaramkondan after it. He also assumed some other titles, the most famous being
Mudikondan, Gangaikondan and Pandita Cholan.

• He was succeeded by his son, Rajadhiraja I.

Q.15) Solution (b)

Explanation:

• The most famous of all Chauhans, is popularly known as Prithviraj Chauhan also known
as Rai Pithora, was one of the most famous kings of Chauhans of Ajmer.

• He ascended to the throne at the tender age of 11, after death of his father
Someshwara, but took command of the administration at the age of 16.

• His renowned Bundelkhand expedition against the Chandella king resulted in the deaths
of two notable Chandella warriors, Alha and Udal.

• He defeated Mohammad Ghori in the First Battle of Tarain (c. 1191 CE).
.
IASBABA’S PRELIMS TEST SERIES (AIPTS) 2024 – TEST 5 (MEDIEVAL HISTORY)

• His court poet, Chanbardai, wrote two poems, Prithviraj Raso and Prithviraj Vijaya.

Q.16) Solution (a)

Explanation:

• Chandellas of bundelkhand trace their descent to a mythical ancestor named


Chandratreya, born of the moon (Chandravanshi lineage of Rajputs). Chandellas belong
to Rajput Dynasty and ruled over central India for approximately 500 years between the
9th and the 13th centuries. As they dominated most of Bundelkhand, they were also
known as the Chandelas of Jejakabhukti (now called Bundelkhand). Initially, they were
feudatories of the Gurjara Pratiharas of Kannauj but became independent rulers in the
10th century. (Hence statement 1 is correct).

• After Yashovarman, Dhanga succeeded to the throne and became the most powerful
king of north India. He was a great conqueror and ruler. During his reign, the empire
reached its pinnacle of power. He was the first independent Chandella king who took
the title of Maharajadhiraja. He seized the Kalinjar fort, occupied the Gwalior fort, and
brought laurels to his dynasty, according to epigraphic documents. (Hence statement 2
is correct).

• Chandella are well known for their art and architecture, most notably for the fully
developed Nagara-style temples at their original capital Khajuraho. During the reigns
of Chandella rulers Yashovarman, Dhanga and yidhyadhara; the Lakshmana temple, the
Vishvanatha Temple and the Kandariya Mahadeva temple were constructed
respectively. (Hence statement 3 is incorrect).

Q.17) Solution (c)

Explanation:

• Kandariya Mahadeva is a famous Hindu temple dedicated to Lord Shiva, located in the
city of Khajuraho, in the Indian state of Madhya Pradesh. The temple was built during
the reign of Dhangadeva of the Chandela dynasty.

• The Kandariya Mahadeva temple is renowned for its intricate and ornate carvings, which
adorn its exterior walls and interior sanctums. The carvings depict various Hindu deities,
celestial beings, animals, and human figures in various poses and activities.
.
IASBABA’S PRELIMS TEST SERIES (AIPTS) 2024 – TEST 5 (MEDIEVAL HISTORY)

• The temple is constructed in the Nagara style of temple architecture, and is built on a
high platform with a series of steps leading up to the entrance. The temple's main shrine
houses a lingam, the phallic symbol of Lord Shiva, and is surrounded by smaller shrines
dedicated to other deities.

• Kandariya MahadevaTemple is considered one of the most beautiful and best-


preserved temples of the Khajuraho group of temples, which were declared a UNESCO
World Heritage Site in 1986. The temple is a popular tourist attraction and is visited by
people from all over the world who come to marvel at its exquisite art and architecture.
(Hence option c) is correct)

Q.18) Solution (b)

Explanation:

• Paramaras were successors of the Rashtrakutas, who controlled Malwa from the early
ninth century with their capital at Dhara (modern Dhar, M.P.). Sindhuraja’s son and
Munja’s nephew Bhoja, who is the most celebrated ruler of the paramara dynasty, and
under whom the dynasty its zenith. (Hence statement 1 is incorrect).

• Under his reign, the Paramara extended their kingdom from Chittor in the north to
upper Konkan in the south, and from the Sabarmati River in the west to Vidisha in the
east.

• Boja is said to have founded the city of Bhojpur, the belief supported by historical
evidence. Beside the Bhojeshwara temple there the construction of three breached
dams in that area attributed to him. This reservoir was formed through construction of 3
earth and stone dams during Bhoja’s reign. (Hence statement 2 is correct).
.
IASBABA’S PRELIMS TEST SERIES (AIPTS) 2024 – TEST 5 (MEDIEVAL HISTORY)

• The first dam, built on Betwa River, trapped the river waters in the depression
surrounded by Hills. Second Dam was constructed in gap between the hills near present
day Mendua village. Third dam, Syclopian dam located in present day Bhopal, diverted
more water from the smaller Kaliasot River into the Betwa Dam Reservoir. The man
made reservior exist until 15th century, when Hoshang shah emptied the lake by
breaching two of the dams.

• He is considered a righteous king who himself was a scholar as well as very


knowledgeable person and wrote many books. According to some source, Bhoj had
attained 64 types of siddhis and he wrote 84 texts on all subjects. His writings cover a
wide variety of topics such as grammar, poetry architecture, yoga shastra, science, art,
natyashastra, music, philosophy, political science, etc. (Hence statement 3 is correct)

Q.19) Solution (d)

Explanation:

• Kumarapala, initially born into a Shaivite family, embraced Jainism later in life, as Jain
legends suggest. (Hence statement 1 is correct).

• According to Jain legends, Kumarapala is revered as the last significant royal supporter
of Jainism and a worthy Jain monarch. Under his rule, Jainism flourished in Gujarat.
(Hence statement 2 is correct).

• After his conversion to Jainism, Kumarapala, according to Jain legends, imposed bans on
animal slaughter, alcohol, gambling, and adultery, signifying his adherence to Jain
principles.

• Reflecting his religious beliefs, Kumarapala undertook the construction of various


temples, encompassing both Brahminical and Jain temples. (Hence statement 3 is
correct).

• Jain scriptures indicate that he was responsible for the construction of numerous
temples in his capital city, Anahilapataka (modern Patan).

Q.20) Solution (d)

Explanation:
.
IASBABA’S PRELIMS TEST SERIES (AIPTS) 2024 – TEST 5 (MEDIEVAL HISTORY)

• Rani ki vav is a magnificent stepwell situated in the town of Patan in the western Indian
state of Gujarat. It was built during the 11th century CE by Queen Udayamati in
memory of her husband, King Bhimdev I of the Solanki dynasty. (Hence statement 1 is
correct).

• The vav, or stepwell, is renowned for its intricate carvings and grand architecture. The
walls of the stepwell are adorned with sculptures depicting various Hindu deities, as
well as scenes from the life of Lord Vishnu. The structure is divided into seven levels,
with each level adorned with impressive carvings and sculptures. (Hence statement 2 is
correct).

• Rani ki vav was declared a UNESCO World Heritage site in 2014, and it is considered one
of the most beautiful and historically significant stepwells in India. It is also a popular
tourist attraction, drawing visitors from around the world who come to marvel at its
beauty and learn about the rich history and culture of the region. (Hence statement 3 is
correct).

Q.21) Solution (b)

Explanation:

The famous battle of Peshawar (1002 CE) took place between:

• Battle of Peshawar, was fought on 27 November 1001 between the Ghaznavid army of
Mahmud of Ghazni and the Hindu Shahi army of Jayapala, near Peshawar.

• Jayapala was defeated and captured, and as a result of the humiliation of the defeat, he
later immolated himself in a funeral pyre.

• This is the first of many major battles in the expansion of the Ghaznavid Empire into the
Indian subcontinent by Mahmud. (Hence option (b) is correct)

Q.22) Solution (c)

Explanation:

• Anangapala II: Anangapala II was a ruler of the Tomara dynasty, which ruled parts of
present-day Delhi and Haryana in India from the 9th to the 12th centuries. Anangapala II
is known for his military campaigns against the Chauhan Rajputs, who were expanding
.
IASBABA’S PRELIMS TEST SERIES (AIPTS) 2024 – TEST 5 (MEDIEVAL HISTORY)

their territories in northern India. He was the founder of the Citadel of Lal Kot in the
Mehrauli area and also built tank known as the Anang Tal. (Hence pair 1 is correctly
matched).

• Jaichandra: Jaichandra who was the last great king of the Ghadavala dynasty, which
ruled parts of present-day Uttar Pradesh in India from the 10th to the 12th centuries.
Jaichandra is known for his conflict with the Chauhan Rajputs and his role in the rise of
Prithviraj Chauhan, one of the most famous Rajput kings. (Hence pair 2 is correctly
matched).

• Avantivarman: He was the founder and a famous king of the Utpala dynasty, which
ruled parts of present-day Kashmir in India. He was known as a great administrator. And
he is known for his patronage of the arts and his contributions to the development of
Kashmiri literature and culture. (Hence pair 3 is correctly matched)

Q.23) Solution (b)

Explanation:

• Iltutmish was a slave-turned-sultan, the third ruler of the Delhi Sultanate (1211-1236
CE). He is known for his administrative and military reforms, which helped to
consolidate the power of the Sultanate.

• One of the key features of Iltutmish's administration was the use of slave soldiers or
bandagans. These were elite military slaves who were trained and maintained by the
state, and were given prominent political offices as a reward for their loyalty and
service. Iltutmish relied heavily on the bandagans to maintain law and order and to
suppress rebellions. (Hence statement 1 is correct).

• Religious Tolerance: Iltutmish displayed religious tolerance during his rule. Despite
being a Muslim ruler, he allowed freedom of worship for Hindus and other religious
communities, fostering a sense of harmony and inclusivity. (Hence statement 2 is
correct).

Important Points/Value Add:

• Expansion of Territory: Iltutmish embarked on military campaigns and successfully


expanded the boundaries of the Delhi Sultanate. He annexed territories in regions such
as Multan, Bengal, and parts of Rajasthan, significantly enlarging his empire.
.
IASBABA’S PRELIMS TEST SERIES (AIPTS) 2024 – TEST 5 (MEDIEVAL HISTORY)

• Patronage of Art and Architecture: Iltutmish was a patron of art and architecture. He
commissioned the construction of notable architectural structures, including the Qutub
Minar in Delhi, which remains an iconic landmark today.

• Iltutmish's achievements as a ruler played a crucial role in strengthening the Delhi


Sultanate and laying the groundwork for subsequent rulers. His administrative reforms,
military successes, economic policies, and cultural patronage left a lasting impact on the
history of medieval India.

• Iltutmish also introduced new coins, including the copper and silver tankas, which
helped to standardize the currency and facilitate trade and commerce. These coins were
widely accepted and used throughout the Sultanate and beyond.

Q.24) Solution (b)

Explanation:

• Diwan-i-Arz in Delhi Sultanate was established by Balban. Diwan-i-Arz was essentially


the Department of Military managed by Ariz-i-Mamalik in order to deal with internal
disturbances and also to check the progression of the Mongols. He was accountable for
the regulation and preservation of the royal army. (Hence statement 1 is incorrect)

• Dagh and Chehra were two important military reforms introduced by Alauddin Khilji,
who ruled the Delhi Sultanate from 1296 to 1316. (Hence statement 2 is correct).

o The Dagh was a branding system for horses used in the army. Under this system,
all horses used in the army were branded with a hot iron stamp that indicated
that they belonged to the state. This ensured that only good quality horses were
used in the army, and that they were properly maintained and cared for.

o The Chehra, on the other hand, was a descriptive role of each soldier. Under this
system, each soldier was assigned a specific role and was identified by a
distinctive mark on his face or forehead. This helped to identify soldiers in battle,
and also helped to keep track of their performance and ensure that they were
properly trained.

• The Asan-e-Mandi system introduced by Alauddin Khilji reflected his efforts to regulate
the economy, control inflation, and ensure social welfare. While its long-term
effectiveness and impact are debated, the system stands as an important example of
economic regulation during the Delhi Sultanate. (Hence statement 3 is incorrect)
.
IASBABA’S PRELIMS TEST SERIES (AIPTS) 2024 – TEST 5 (MEDIEVAL HISTORY)

Q.25) Solution (a)

Explanation:

• The Iqtadari system was an administration system that evolved during the Delhi Sultan
Iltutmish. Iqta system was first introduced by Muhammad of Ghur in 1206 CE but the
institutional form was given by Iltutmish. (Hence statement 1 is correct).

• Under this system, the kingdom was divided into large and small tracts of land known as
Iqtas.

• These Iqtas were assigned to various people such as Officers, Nobles, and Soldiers to
ease the flow of revenue collection and to lead towards flawless administration. (Hence
statement 2 is correct).

• The Iqtadars can transfer their Iqtas from one region to another region every 3-4 years.

• Small Iqtas were held by Individuals and the heads in charge were not given any
administrative responsibilities.

• During the Delhi Sultanate period, the Iqta system underwent numerous changes.

• In the initial stages, Iqta was a revenue yielding piece of land.

• During the reign of Firuz Shah Tughlaq, central control over Iqtas got diluted and the
Iqtas became hereditary. (Hence statement 3 is incorrect)

Q.26) Solution (d)

Explanation:

• Ghiyasuddin Balban was a powerful ruler of the Delhi Sultanate who ruled from 1266 to
1287. He was a slave who rose to power under the patronage of Sultan Iltutmish and
became the most influential courtier during the reign of his successor, Razia Sultana.
After the death of Razia, the Delhi Sultanate faced political instability, and Balban took
advantage of the situation and ascended to the throne.

• He was deeply racist and excluded non-Turks from administration. He stood forth as the
champion of Turkish nobility. Indian Muslims were not given important posts in the
government. (Hence statement 1 is incorrect)
.
IASBABA’S PRELIMS TEST SERIES (AIPTS) 2024 – TEST 5 (MEDIEVAL HISTORY)

• The Chahalgani were introduced during the reign of Iltutmish, the third ruler of the
Delhi Sultanate in India. The Chahalgani had a significant say in the appointment of
officials and played a crucial role in the succession of the throne. Over time, the
Chahalgani became more powerful and started to control the Sultanate's affairs, which
led to political instability and weakened the central authority. (Hence statement 2 is
incorrect)

• Balban, tried to break the power of the Chahalgani by creating a new group of loyal
officers called the Barids. He also introduced a spy system, where spies were appointed
to keep a check on the nobles. Balban's policies helped to centralize power in the hands
of the Sultan and reduce the influence of the nobles.

• Balban emphasized on the ceremony of Sajda and Paibos as a symbol of his power and
authority. He insisted that all courtiers and visitors should prostrate themselves before
him and touch his feet as a sign of respect. This was a way of emphasizing the
superiority of the ruler and the subordination of his subjects. (Hence statement 3 is
correct)

Q.27) Solution (d)

Explanation:

• Amir Khusrau (1253-1325) was a renowned poet, musician, and scholar who lived in the
13th century in the Delhi Sultanate. His real name was Abul Hasan. He was one of the
few Indian writers of Persian poetry whose works have been read and admired beyond
their own country. His works represent the beginning of a new trend in India based
Persian literature – the trend of the growing familiarity with the Indian literature and
influence of the Indian literature on the Persian writings in India.

• He was the Indian born son of a Turkish immigrant. He began his career as a courtier
and poet during the reign of Sultan Balban. He became a disciple of Shaikh Nizammudin
Auliya, the famous Sufi saint of the Chisti order.

• He lived through the reigns of seven Sultans of Delhi (Balban, Muhammad, Cakubad,
Jalaluddin Khilji, Alauddin Khilji, Mubarak Shah Khaliji, Gaisuddin Tughlaq) and was
connected with their courts.
.
IASBABA’S PRELIMS TEST SERIES (AIPTS) 2024 – TEST 5 (MEDIEVAL HISTORY)

• Khusrau served under several rulers of the Delhi Sultanate, including Balban, Alauddin
Khalji, Qutbuddin Mubarak Shah, and Ghiyasuddin Tughlaq. He was particularly close
to Alauddin Khalji and Muhammad bin Tughlaq. (Hence statement 1 is correct)

• He created a new style of Persian, which came to be known as Sabaq-i-Hindi (style of


India). He is credited with creating a new style of Persian that combined languages such
as Hindi and Arabic with the classical Persian language. (Hence statement 2 is correct)

• Masnavis are narrative poems, which have great literary and historical value, which
have been written by Khusrau. Khusrau is considered one of the greatest poets of the
Persian and Urdu languages. He wrote many masnavis, which are narrative poems in
rhyming couplets, including the famous "Matla-ul-Anwar" and "Qiran-us-Sa'dain". These
works have both literary and historical value as they provide insights into the cultural
and social life of medieval India. (Hence statement 3 is correct)

Important Points/Value Add:

• In addition to his poetic contributions, Khusrau is also recognized for his musical
innovations. He is credited with creating new musical instruments, developing new
melodic structures, and pioneering the musical genre of Qawwali.

• Amir Khusrau's works have had a lasting impact on Indian literature, music, and culture.
His poetry continues to be celebrated and recited, and his musical contributions have
influenced generations of musicians. Khusrau's legacy as a polymath, blending poetry,
music, and scholarship, has made him an iconic figure in the cultural history of India.

Q.28) Solution (b)

Explanation:

Important Concept/Term and their meaning used during Delhi Sultanate

• Amir-i-Behr: Controller of boats

• Mameluks: Slave officers

• Rai Rayans: The title given by Alauddin Khalji to Rama Deva of Devagir

• Sarai Adl – Cloth Market

• Mandi – Grain Market


.
IASBABA’S PRELIMS TEST SERIES (AIPTS) 2024 – TEST 5 (MEDIEVAL HISTORY)

• Munhiyans – Secret spies

• Naib – Deputy Assistants

• Paibos – Kissing of feet’s

• Tanka – Silver coin of Delhi Sultanate

Q.29) Solution (a)

Explanation:

• Ibn Battuta (1304-1368/1369) was a famous Moroccan explorer and scholar who is
widely recognized for his extensive travels. He embarked on a remarkable journey that
spanned over three decades and covered around 75,000 miles, making him one of the
most prolific travelers in history.

• Throughout his travels, Ibn Battuta encountered various cultures, societies, and rulers.
He documented his experiences in his travelogue called "Rihla," he written it in the
Arabic language, which provides valuable insights into the diverse civilizations and
customs of the regions he explored. His writings are not only significant for their
geographical and historical information but also for their vivid descriptions of people,
landscapes, and social dynamics. (Hence statement 1 is correct)

• He came to India during the tenure of Muhammad Bin Tughlaq. The Sultan was
impressed by his scholarship, and appointed him the qazi or judge of Delhi. He remained
in that position for several years, until he fell out of favour and was thrown into prison.
Once the misunderstanding between him and the Sultan was cleared, he was restored
to imperial service, and was ordered in 1342 to proceed to China as the Sultan’s envoy
to the Mongol ruler. (Hence statement 2 is correct)

• Ibn Battuta in his book has also given the details of the prevalence of robbery during his
travel in different parts of the country. He himself was attacked several times and
many of his fellow survivors also lost their lives. While travelling from Multan to Delhi,
for instance, his caravan was attacked and many of his fellow travellers lost their lives;
those travellers, who survived, including Ibn Battuta, were severely wounded. (Hence
statement 3 is incorrect)

Q.30) Solution (d)


.
IASBABA’S PRELIMS TEST SERIES (AIPTS) 2024 – TEST 5 (MEDIEVAL HISTORY)

Explanation:

• Shahna-i-Mandi was an important official appointed by Alauddin Khilji, who ruled the
Delhi Sultanate from 1296 to 1316.

• The term "Shahna-i-Mandi" means "Controller of the Market" in Persian.

• Shahna-i-Mandi was responsible for implementing Alauddin Khilji's market regulation


policies, which aimed to control the prices of essential commodities such as grain and
cloth.

• Under Alauddin Khilji's market regulation system, the state would purchase these
commodities at a fixed price and sell them to the public at a subsidized rate.

• Shahna-i-Mandi was responsible for overseeing this process and ensuring that the
commodities were purchased and sold at the right price. He was also responsible for
managing the markets and ensuring that there was no hoarding or black marketing.

• Shahna-i-Mandi played an important role in the administration of the Delhi Sultanate


during the reign of Alauddin Khilji. His efficient management of the market regulation
system helped to stabilize the economy and ensure that the common people had access
to basic necessities.

Q.31) Solution (b)

Explanation:

During the medieval period in India, the Delhi Sultanate established a well-organized
administrative structure to effectively govern its vast territories. The administration consisted
of different departments of state, each with specific functions and responsibilities. Here is a
brief overview of the main departments and their functions:

• Diwan-i-Qaza: was the department responsible for the administration of justice.

• Diwan-i-kohi: In charge of managing the agricultural activities in the kingdom. (Hence


pair 3 is correctly matched)

• Diwan-i-Wazarat (Department of Finance): This department was responsible for


managing the finances of the Sultanate. It collected revenue through taxes, maintained
financial records, allocated funds for various expenses, and ensured the smooth
functioning of the treasury.
.
IASBABA’S PRELIMS TEST SERIES (AIPTS) 2024 – TEST 5 (MEDIEVAL HISTORY)

• Diwan-i-Ariz (Department of Military Affairs): The Diwan-i-Ariz was in charge of the


Sultanate's military affairs. It oversaw the recruitment and training of soldiers, organized
the army, managed weapons and supplies, and coordinated military campaigns.

• Diwan-i-Insha (Department of Correspondence): This department handled all official


correspondence and maintained records and archives. It facilitated communication
between different departments, as well as with other rulers and foreign dignitaries.

• Diwan-i-Rasalat (Department of Religious Affairs): The Diwan-i-Rasalat dealt with


matters related to religion and Islamic law. It included the appointment of Qazis
(judges), overseeing the enforcement of Sharia (Islamic law), and ensuring the
functioning of mosques and religious institutions. (Hence pair 2 is correctly matched)

• Diwan-i-Siyasat (Department of Political Affairs): The Diwan-i-Siyasat was responsible for


diplomatic affairs, foreign relations, and political negotiations. It managed interactions
with neighboring regions, maintained alliances, and handled matters of state security
and intelligence. (Hence pair 1 is incorrectly matched)

Important Points/Value Add:

• Diwan-i-Mustakhraj (Department of Revenue): This department was responsible for


assessing and collecting revenue from different sources, including land, agriculture,
trade, and customs. It ensured efficient taxation and revenue generation for the
Sultanate.

• Diwan-i-Ishaat (Department of Public Information): The Diwan-i-Ishaat was involved in


disseminating official information and maintaining public communication. It included
tasks such as issuing royal proclamations, announcing policies, and promoting cultural
and literary activities.

• Diwan-i-Jagir (Department of Land Grants): The Diwan-i-Jagir managed the distribution


of land grants (jagirs) to nobles and officials in exchange for their service. It maintained
land records, assessed the value of the grants, and ensured their proper allocation.

Q.32) Solution (a)

Explanation:

• During Muhammad-Bin-Tughlaq's reign, a separate department called Diwan-i-amir-i-


kohi was established to look after the welfare of the farmers. This department was
.
IASBABA’S PRELIMS TEST SERIES (AIPTS) 2024 – TEST 5 (MEDIEVAL HISTORY)

responsible for implementing several agricultural reforms, such as the construction of


canals, wells, and dams to improve irrigation, and the development of new lands for
agriculture. The department also provided loans to farmers at low-interest rates to help
them overcome the difficulties caused by natural disasters or droughts. (Hence
statement 1 is correct.)

• The Taccavi loans, provided by the Diwan-i-amir-i-kohi, were introduced to assist


farmers during challenging times. However, some officials misused these loans, and the
process of recovering them was inefficient, resulting in a significant burden on the
treasury.

• Nevertheless, the policy advocated by Muhammad Tughlaq to expand and improve


cultivation was not forgotten. Firuz continued this approach and it was further
enhanced later on during the reign of Akbar. (Hence statement 2 is incorrect.)

Other reforms by Muhammad-Bin-Tughlaq

• Shifting of the Capital: One of the most notable and controversial reforms was the
shifting of the capital from Delhi to Daulatabad (present-day Maharashtra). Muhammad
bin Tughlaq believed that the new location would be strategically advantageous.
However, this forced migration of the population resulted in immense hardship, loss of
life, and ultimately, the abandonment of the plan.

• Introduction of Token Currency: In an attempt to streamline the monetary system,


Muhammad bin Tughlaq introduced copper coins as token currency. However, due to
insufficient supply and lack of confidence in the new currency, this reform failed and
caused economic instability.

• Land Reforms: Muhammad bin Tughlaq implemented land reforms aimed at reducing
the influence of the nobles and centralizing control over agricultural lands. He
introduced the system of measurement known as "dohri" to assess land revenue.
However, these reforms faced resistance and created discontent among the nobility and
peasants.

• Administrative Reforms: Muhammad bin Tughlaq attempted to strengthen the


administration by expanding the bureaucracy and implementing a system of checks and
balances. He appointed regional governors called "naibs" to improve governance.
However, these reforms were challenging to implement effectively, leading to
administrative inefficiencies and corruption.
.
IASBABA’S PRELIMS TEST SERIES (AIPTS) 2024 – TEST 5 (MEDIEVAL HISTORY)

• Military Reforms: Muhammad bin Tughlaq sought to strengthen the military by


employing mercenaries and introducing new weapons and technologies. He also
established a separate department for military affairs. However, these reforms faced
logistical challenges and did not significantly enhance the military capabilities of the
Sultanate.

• Muhammad bin Tughlaq's reforms were characterized by their ambitious nature, but
many of them were poorly executed and had adverse consequences. Despite the
failures, his reign also witnessed advancements in various fields such as literature, art,
and architecture. Overall, his reforms left a significant impact on the administration and
history of the Delhi Sultanate.

Q.33) Solution (d)

Explanation:

"Padmavat" is a medieval Indian epic poem written in the Awadhi language by the poet Malik
Muhammad Jayasi in the 16th century. He completed his work Padmavat during Sher Shah's
reign. The poem tells the story of the legendary queen Padmini of Chittorgarh and her husband,
King Ratan Singh, and their battle against the invading Muslim ruler Alauddin Khalji. The poem
is known for its vivid descriptions, poetic language, and romanticism.

Q.34) Solution (b)

Explanation:

Firoz Shah Tughlaq, the Sultan of Delhi from 1351 to 1388, founded several cities during his
reign. Some of the cities founded by him are:

• Firozabad - It served as his capital during his reign. The city was located near Delhi next
to the river Yamuna. It was known for its impressive architecture, including a palace, a
mosque, and a madrasa.

• Jaunpur - located in present-day Uttar Pradesh. Initially Jaunpur was a major center of
Sufi and Urdu knowledge and culture. As years passed by, Jaunpur slowly and steadily
flourished and prospered as an important cultural center. The city served as an
important center of learning and culture, and it was known for its magnificent mosques
and other public buildings.
.
IASBABA’S PRELIMS TEST SERIES (AIPTS) 2024 – TEST 5 (MEDIEVAL HISTORY)

• Hissar Firoza - ‘Hisar’ is an Arabic word which means ‘Fort’, located in present-day
Haryana. The city was known for its fort, inside the fort a fine palace, having a complex
of different buildings was built. Well laid out gardens added to the beauty of the palace.

• Fatehabad – He named it after his son Fateh Khan as Fath. Located in present-day
Haryana. The city was known for its beautiful buildings and gardens, and it served as an
important center of trade and commerce.

• These are some of the cities founded by Firoz Shah Tughlaq during his reign.

• Faridabad City which was established in AD 1707 by Sheikh Farid, the treasurer of
Jahangir with the objective of protecting the highway passed through the city. Sheikh
Farid built a fort, a tank and a mosque. (Hence city 3 is incorrect)

Q.35) Solution (b)

Explanation:

• Tahkik-i-Hind (Reality of Hindustan)- in this book Alberuni gives an account of the


religious, literary and scientific traditions of India. Occasionally he also gives a
description of the roads, gives, rivers, weights and measure, Currency, etc. His book
consists of eighty chapters and investigates the truth about the contemporary India life.
Their religious and philosophical doctrines were discussed in an impartial manner.
Alberuni tells us that the Hindus were excellent philosophers good mathematicians and
astronomers. (Hence statement 1 is correct)
.
IASBABA’S PRELIMS TEST SERIES (AIPTS) 2024 – TEST 5 (MEDIEVAL HISTORY)

• The two most well-known works of Ziauddin Barani are Tarikh-i-Firozshashai and
Fatawa-i- Jahandari, which provide valuable information about the History of that
period.

• He completed his work ―Tarikh-i- Firozshahi in 1357 and dedicated the same to the
ruling Sovereign. It may be noted that the book does not exclusively deal with the reign
of Firozshah. It begins with Balban and ends with the sixth regnal year of Firozshah.
(Hence statement 2 is incorrect).

• Fatawa-i-Jahandari describes certain ethics of administration and important ideas of


government. It begins with an account of the creation of world and the prophets and
their teachings .it analyses the good qualities, virtues and talents that a monarch should
possess. (Hence statement 3 is correct).

Q.36) Solution (b)

Explanation:

Two technological and stylistic developments are noticeable from the twelfth century.

• One of them is that the weight of the superstructure above the doors and windows was
sometimes carried by arches. This architectural form was called “arcuate”.
.
IASBABA’S PRELIMS TEST SERIES (AIPTS) 2024 – TEST 5 (MEDIEVAL HISTORY)

• Second, Limestone cement was increasingly used in construction. This was very high-
quality cement, which, when mixed with stone chips hardened into concrete. This made
construction of large structures easier and faster. (Hence Statement 1 is correct).

• During the medieval period in India, Hindu rulers were often the biggest patrons of
temple construction. This was because temples were seen as important symbols of
political and religious power, and the construction of grand temples was seen as a way
for rulers to demonstrate their wealth and influence. Many Hindu rulers, such as the
Cholas, the Vijayanagar Empire, and the Hoysalas, were known for their patronage of
temple construction. They commissioned elaborate temple complexes that were
adorned with intricate carvings, sculptures, and paintings, and which often housed
important religious artifacts and idols. In addition to being symbols of political and
religious power, temples were also important centers of economic activity. They served
as centers of trade and commerce, and provided employment to a large number of
people, including architects, artisans, and priests. (Hence statement 2 is correct).

• Jama Masjid also known as the Masjid-i-Jehan-Numa built by Shah Jahan in his new
capital at Shahjahanabad, Delhi. (Hence statement 3 is incorrect).

• Babur described in his autobiography, his interest in planning and laying out formal
gardens, placed within rectangular walled enclosures and divided into four quarters by
artificial channels. These gardens were called chahar bagh, four gardens, because of
their symmetrical division into quarters. Beginning with Akbar, some of the most
beautiful chahar baghs were constructed by Jahangir and Shah Jahan in Kashmir, Agra
and Delhi. (Hence statement 4 is correct).

Q.37) Solution (b)

Explanation:

• The Amara-Nayaka system was a major political innovation of the Vijayanagar Empire. It
is likely that many features of this system were derived from the Iqta system of the
Delhi Sultanate. (Hence statement 1 is correct).

• The Amara-Nayakas were military commanders, who were given territories to govern by
the Raya. They collected taxes and other dues from the peasants, crafts persons and
traders in the area. They retained part of the revenue for personal use and for
maintaining a stipulated contingent of horses and elephants. These contingents
.
IASBABA’S PRELIMS TEST SERIES (AIPTS) 2024 – TEST 5 (MEDIEVAL HISTORY)

provided the Vijayanagar kings with an effective fighting force, with which they brought
the entire southern peninsula under their control. (Hence statement 2 is correct).

Q.38) Solution (a)

Explanation:

• Muhammad-Bin-Tughlaq was a ruler of the Delhi Sultanate in the 14th century. He


implemented several monetary reforms during his reign, including introducing a copper
coin with the same value as the silver tanka. This move was an attempt to increase the
availability of currency and stabilize the economy. However, the copper coins were
quickly counterfeited, leading to inflation and economic instability. (Hence statement 1
is correct)

• The introduction of token currency, which means that coins were minted with a lower
intrinsic value than their face value. This was done to increase the supply of currency
without using valuable metals like gold or silver. While it is true that Muhammad-Bin-
Tughlaq introduced token currency, it is not accurate to say that it was the first time
ever in world history. Qublai Khan of China had already successfully experimented
with a token currency. A Mongol ruler of Iran, Ghazan Khan, had also experimented
with it. Muhammad Tughlaq decided to introduce a bronze coin which was to have the
same value as the silver tanka. Specimen of this coin has been found in different parts of
India. The idea of a token currency was a new one in India and it was difficult to
induce the traders, as well as the common man to accept it. (Hence statement 2 is
incorrect)

Q.39) Solution (a)

Explanation:

• Muhammad bin Tughlaq (1325-1351) was a very attractive character in the history of
medieval India owing to his ambitious schemes and novel experiments. He was very
tolerant in religious matters. (Hence statement 1 is correct)

• Contemporary writers like Isami, Barani and Ibn Battutah were unable to give a correct
picture about Tughlaq’s personality. But, Muhammad bin Tughlaq was the only Delhi
Sultan who had received a comprehensive literary, religious and philosophical
education. (Hence statement 2 is correct)
.
IASBABA’S PRELIMS TEST SERIES (AIPTS) 2024 – TEST 5 (MEDIEVAL HISTORY)

• Tughlaq maintained diplomatic relations with far off countries like Egypt, China and
Iran. He also introduced many liberal and beneficial reforms. But all his reforms failed.
(Hence statement 3 is incorrect).

Q.40) Solution (b)

Explanation:

• The Kudirai Chettis were a local community of horse merchants. Warfare during the
Vijayanagar times depended upon effective cavalry, and the import of horses from
Arabia and Central Asia was very important for rival kingdoms. This trade was initially
controlled by the Arab traders. Local communities of merchants known as Kudirai
Chettis or horse merchants, also participated in these exchanges. (Hence statement 1 is
incorrect).

• The Vijayanagara Empire was known for its innovative and efficient urban planning. One
of the unique features of their cities was the inclusion of agricultural tracts within the
fortified walls. This ensured that the city was self-sufficient in terms of food production
and could withstand sieges for longer periods. (Hence statement 2 is correct).

• Abdur Razzaq, an ambassador sent by the ruler of Persia to Calicut in the fifteenth
century, was greatly impressed by the fortifications and mentioned seven lines of forts.
He points that what was most significant about this fortification is that it enclosed
agricultural tracts. He noted that

• One of the most prominent waterworks, to be seen among the ruins of Vijayanagar
waterworks is the Hiriya canal. This canal was built by the kings of the Sangama
dynasty. It drew water from a dam across the Tungabhadra and irrigated the cultivated
valley that separated the “sacred center” from the “urban core”. (Hence statement 3 is
incorrect).

Q.41) Solution (c)

Explanation:

The sources for the study of Vijayanagar are varied, such as literary, archaeological and
numismatics.
.
IASBABA’S PRELIMS TEST SERIES (AIPTS) 2024 – TEST 5 (MEDIEVAL HISTORY)

• Krishnadevaraya’s Amukthamalyada, Gangadevi’s Maduravijayam and Allasani


Peddanna’s Manucharitam are some of the indigenous literature of this period. Many
foreign travellers visited the Vijayanagar Empire and their accounts are also valuable.

• The Moroccan traveller, Ibn Battuta, Venetian traveller Nicolo de Conti, Persian traveller
Abdur Razzaq and the Portuguese traveller Domingo Paes were among these who left
valuable accounts on the socio-economic conditions of the Vijayanagar Empire.

• Ratnavali is a Sanskrit drama about a beautiful princess named Ratnavali and a great
king named Udayana. It is attributed to the Indian emperor Harshavardhana, and does
not pertain to the Vijayanagara Empire.

• The text Kavyamimamsa is a work that serves as a practical guide for poets, written
during 880–920 AD. It is written by Rajashekhara, who was a court poet of the Pratihara
dynasty. Hence it is also not related to Vijayanagara Empire.

Q.42) Solution (c)

Explanation:

• Krishnadevarya built a new city, called Nagalapuram. He was the greatest of the
Vijayanagar rulers and belonged to the Tuluva dynasty. He also built the famous
Vittalaswamy and Hazara Ramaswamy temples at Vijayanagar. Besides, he built a large
number of Rayagopurams. (Hence statement 1 is incorrect).

• Krishnadevaraya was a great patron of literature and art, and he was known as Andhra
Bhoja. The peak of literary achievement was reached during his reign. He himself was a
scholar in Sanskrit and Telugu. He authored a Telugu work, Amukthamalyadha and
Sanskrit works, Jambavati Kalyanam and Ushaparinayam. (Hence statement 2 is
correct).

• Foreign travellers have left vivid accounts on the splendor of buildings and luxurious
social life in the city of Vijayanagar. Nicolo de Conti refers to the prevalence of slavery.
(Hence statement 3 is correct).

• Domingo Paes mentions of the beautiful houses of the rich and the large number of
their household servants.
.
IASBABA’S PRELIMS TEST SERIES (AIPTS) 2024 – TEST 5 (MEDIEVAL HISTORY)

Q.43) Solution (a)

Explanation:

• Firuz Shah Bahmani was well-acquainted with the religious sciences, that is,
commentaries on the Quran, jurisprudence, etc., and was particularly fond of the
natural sciences, such as botany, geometry, logic, etc. He was a good calligraphist and a
poet, and often composed extempore verses.

• According to Ferishta (traveller), he was well versed not only in Persian, Arabic and
Turkish, but also in Telugu, Kannada and Marathi. He had read the Old and the New
Testaments, and respected the tenets of all religions.

• The most remarkable step taken by Firuz Shah Bahmani was the induction of the Hindus
in the administration on a large scale. It is said that from his time, the Deccani Brahmans
became dominant in the administration, particularly in the revenue administration. The
Deccani Hindus also provided a balance against the influx of the foreigners. (Hence
statement 2 is correct).

• Firuz Shah Bahmani was a patron of arts and sciences. He encouraged the pursuit of
astronomy and built an observatory near Daulatabad in present-day Maharashtra.
(Hence statement 1 is correct).

• He paid much attention to the principal ports of his kingdom, Chaul and Dabhol, which
attracted trading ships from the Persian Gulf and the Red Sea, and brought in luxury
goods from all parts of the world.

• It was Ahmad Shah Wali who transferred Bahmani capital to Bidar. (Hence statement 3
is incorrect).

Q.44) Solution (a)

Explanation:

• The term bargir referred to cavalrymen in Maratha and Mughal armies.

• The word comes from the Persian “bargir”, literally meaning “burden taker”

• But in the Mughal and Maratha armies, the term signified a soldier who rode a horse
furnished by his employer.
.
IASBABA’S PRELIMS TEST SERIES (AIPTS) 2024 – TEST 5 (MEDIEVAL HISTORY)

• In the Maratha cavalry, any able-bodied person could enlist as a bargir, unless he had
the means to buy a horse and military outfit.

Q.45) Solution (b)

Explanation:

• During the rule of Bahmani kingdom, there were two groups in the ranks of amirs: one
was the Deccan’s who were immigrant Muslims and the other was the Afaqis or
Pardesis who had come from Central Asia, Iran, and Iraq, and had settled in the
Deccan region. (Hence option (b) is correct)

• There was a formidable rivalry between the Afaqis and the Dakhnis did not allow the
groups to come together, which resulted in them retaining their separate identities. The
Bahmani sultans were sunni in faith, while under the growing influence of the Afaqis the
five successor states mostly followed Shiism

Q.46) Solution (a)

Explanation:

• Devaraya II, also known as Praudha Devaraya, was a dominant ruler of the Sangam
dynasty who exhibited his bravery by defeating the Bahmani Sultan while still a prince.
Upon ascending the throne, he embarked on expeditions to expand his empire. (Hence
statement 1 is correct).

• Devaraya II was a patron of art and literature and was himself a scholar. He authored
"Mahanataka Sudhanidhi" and had poets like Gadugina Naranappa (Kumaravyasa),
Chamarasa, and Srinath in his court. During his reign, architecture, handicrafts, music,
and painting thrived, and he constructed the Hazara Ramaswamy temple in Hampi.
(Hence statement 2 is correct).

• The Persian pilgrim Abdul Razak visited Vijayanagara during Devaraya II's reign and
praised it as unparalleled in the world. Devaraya II, also known as Veera
Pratapadevaraya, Maharaya, and Gajabentekara, passed away in 1446. (Hence
statement 3 is correct).
.
IASBABA’S PRELIMS TEST SERIES (AIPTS) 2024 – TEST 5 (MEDIEVAL HISTORY)

• Administration and Governance: Devaraya II implemented administrative reforms to


improve governance and ensure efficient revenue collection. He appointed capable
ministers and officials to various administrative positions.

• Trade and Commerce: Devaraya II promoted trade and commerce within the empire
and with foreign powers. He established commercial relations with the Ming Dynasty of
China and the Arab traders, strengthening economic ties.

• Diplomatic Relations: Devaraya II maintained diplomatic relations with neighboring


kingdoms and the Bahmani Sultanate, balancing diplomacy and military action to
safeguard Vijayanagara's interests.

• Succession and Legacy: Devaraya II was succeeded by his son, Mallikarjuna Raya, after
his death in 1446 CE. His reign is considered a period of stability and prosperity for the
Vijayanagara Empire.

Q.47) Solution (b)

Explanation:

• The Nayankara system was a political system that was used by the Vijayanagara Empire,
a prominent South Indian kingdom that flourished from the 14th to the 16th century.

• This system granted partial independence to the empire's feudatories, who were known
as Nayankaras. Nayankaras were granted land in exchange for their military service to
the king. (Hence statement 1 is correct).

• These feudatories had partial autonomy, but they were still required to pay an annual
tribute to the king and provide a specified number of soldiers, horses, and elephants for
the defense of the empire. (Hence statement 2 is incorrect).

• Under the Nayankara system, there were several types of feudatories, including
Samantas, Nayakas, Dandanayakas, Mandaleshwararajas, and Wodeyars.

• The Nayankaras were required to take the "sovereign oath" at the king's court, which
was a symbolic act that affirmed their loyalty to the empire. (Hence statement 3 is
correct).
.
IASBABA’S PRELIMS TEST SERIES (AIPTS) 2024 – TEST 5 (MEDIEVAL HISTORY)

• Unlike provincial officials, who ruled as representatives of the king, Nayankaras were
military vassals who held land, also known as "amaram."

• The Nayankara system played an important role in the administration and defense of
the Vijayanagara Empire. Nunez states that During the reign of Achyutaraya, the land
was divided among more than 200 chieftains. This system, however, also had its
shortcomings.

• The Nayankaras who were loyal to the Swami under the king disobeyed after the death
of Krishna Devaraya, which led to the empire's eventual downfall. Despite its flaws, the
Nayankara system is considered to be an important part of the Vijayanagara Empire's
history, and it continues to be studied and analyzed by historians and scholars today.

Q.48) Solution (b)

Explanation:

• The Vijayanagara Empire saw the involvement of women in various fields including
warfare and wrestling. Despite the tragic incident of Madigowda and his daughter
Hariyakka who died during wrestling, it became a symbol of female freedom and
courage. (Hence statement 1 is correct).

• Many women practitioners were also noted during this period such as Gangadevi,
Pennaim Jaganohini, Abhirama Kamakshi, and Mohanangi who were educated mainly in
music, dance, drama and instruments. Women also owned land in some places as
evidenced by some inscriptions found in the Vijayanagara Empire.

• However, women were also exploited during this period. The Devadasi system was
prevalent where women were dedicated to the temples and were confined to the royal
court and temples to become the pleasures of the public. This led to the pollution of the
temple atmosphere but had religious acceptance. (Hence statement 2 is correct).

• The practice of Sati was also prevalent among the upper castes where a woman would
be burned alive along with her husband upon his death. The tourists who visited
Vijayanagar during this time also witnessed the Sati rituals. (Hence statement 3 is
incorrect).

Q.49) Solution (d)


.
IASBABA’S PRELIMS TEST SERIES (AIPTS) 2024 – TEST 5 (MEDIEVAL HISTORY)

Explanation:

• The tax system of the Vijayanagara Empire, which existed from the 14th to the 17th
century in South India, played a crucial role in the administration and economy of the
empire. The tax system was designed to ensure efficient revenue collection and provide
the necessary resources for the functioning of the empire.

• In addition to land revenue, the Vijayanagara kings also relied on various other forms of
taxation to generate income. For instance, they collected a Ghari tax based on the size
and value of houses, with two-pana annual revenue for a storeyed house and three
pana for a house with a hall.

• Fishery duty was collected from lakes and reservoirs, while tolls were collected from
festivals held in cities and towns. Feudal lords also had to pay tribute to the emperor,
surrendering half of their income every year. (Hence statement 1 is correct).

• In times of war, the government would levy special contributions on the subjects and
feudal lords. The state treasury also benefited from taxes on professions such as
weavers, miners, barbers, potters, blacksmiths, carpenters, and courtesans, who were
required to submit fixed taxes to the government periodically. (Hence statement 2 is
correct).

• Additionally, the state levied taxes on forest products, salt, and mines, as well as
imposing fines and marriage taxes. (Hence statement 3 is correct).

Q.50) Solution (c)

Explanation:

• The Vijayanagara Empire introduced its own coinage system, distinct from earlier
dynasties. It issued a range of coins made from various metals, including gold, silver,
copper, and alloys. These coins were minted in different denominations to cater to
various economic transactions.

• This paragraph discusses the use of gold, silver, and copper coins during the reign of the
Vijayanagara Empire. The coins feature various images such as Hanuman, Garuda,
elephants, Shiva-Parvati, Lakshminarayana, Saraswati-Brahma, and the royal emblem of
Vijayanagara, Varaha. (Hence statement 1 is correct).
.
IASBABA’S PRELIMS TEST SERIES (AIPTS) 2024 – TEST 5 (MEDIEVAL HISTORY)

• The title Sri Vira Harihara is also found on the coins of Harihara I. Abdul Razak, a traveler
during the second Devarayan period, provided information about the coinage system,
which included coins with images of elephants.

• Private individuals commissioned by the emperor were also able to mint coins;
however, Coins issued by private individuals were in use only in certain areas. (Hence
statement 2 is correct).

• Gadyana coins were issued in Barakur and Mangalore. Harihara I introduced the idea of
paying taxes in cash, which led to an increase in coin production and circulation.
Additionally, the gold bars during Krishnadevaraya's time weighed 52 grams.

Q.51) Solution (c)

Explanation:

• The reign of Jahangir saw an important innovation in the Mansabdari system, his
introduction a new provision in the sawar rank. According to it, a part of sawar rank was
termed Du-Aspah Sih-Aspah in case of select Mansabdars. He probably introduced this
provision to promote nobles of his confidence and strengthen them militarily. (Hence
statement 1 is correct).

• Shahjahan introduced the month-scale in the Mansabdari system to compensate the


gap between Jama (estimated income) and hasil (actual realisation). As the
Mansabdars were generally paid through revenue assignments, i.e., jagirs. The biggest
problem was that calculation was made on the basis of the expected income (Jama)
from the jagir during one year. It was noticed that the actual revenue collection always
fell short for the estimated income. In such a situation the salary was fixed by a method
called month scale. (Hence statement 2 is correct).

• Aurangzeb continued with all this changes and created an additional rank card Mashrut
(conditional). This was an attempt to increase the sawar rank of the Mansabdar
temporarily. Aurangzeb added another deduction called Khurak-i-dawwab, towards
meeting the cost for feed of animals in the Imperial stables. (Hence statement 3 is
correct).

Q.52) Solution (c)


.
IASBABA’S PRELIMS TEST SERIES (AIPTS) 2024 – TEST 5 (MEDIEVAL HISTORY)

Explanation:

• "Zawabit" is an Arabic term that was used during the medieval period of Indian history,
particularly during the Delhi Sultanate and Mughal Empire eras, to refer to legal
principles and rules.

• These legal principles were based on Islamic law, also known as Shariah, and were used
to guide various aspects of life, including personal behavior, family law, commercial
transactions, and criminal justice. The Zawabit were used by the rulers and judges to
administer justice and maintain law and order. (Hence option (c) is correct answer)

• The Zawabit were compiled in various books, such as the Hidayah, which was a
comprehensive manual of Islamic law and principles that was widely used in India during
the medieval period. Other notable works include the Fatawa-i-Alamgiri, which was a
compilation of legal opinions issued by Islamic scholars during the Mughal period.

• Overall, the Zawabit played an important role in shaping the legal system of medieval
India and influenced the development of Islamic law in the region.

Q.53) Solution (c)

Explanation:

Duaspa-Sihaspa:

• Jahangir introduced a system whereby selected nobles could be allowed to maintain a


large quota of troopers, without raising their Zat rank.

• This is called Duaspah-Sihaspah system (literally, trooper with 2 or 3 horses) which


implied that a mansabdar holding this rank has to maintain and was paid for double the
quota of troopers indicated by his Sawar rank.

Q.54) Solution (a)

Explanation:

The Mughal court chronicles were written in Persian. The famous chronicles are Akbar Nama,
Shahjahan Nama and Alamgir Nama. Turkish was the mother tongue of Mughals, but it was
Akbar who made Persian the leading language of Mughal court. Persian became Indianised by
absorbing local idioms. Urdu sprang from the interaction of Persian with Hindavi. All books in
.
IASBABA’S PRELIMS TEST SERIES (AIPTS) 2024 – TEST 5 (MEDIEVAL HISTORY)

Mughal India were handwritten manuscripts and were kept in Kitabkhana . i.e. scriptorium.
(Hence statement 1 is incorrect).

The Mahabharata was translated as the Razmnama (Book of Wars). In 1574, Akbar started a
Maktab Khana or "House of Translation" in his new capital at Fatehpur Sikri. He assigned a
group to translate the Sanskrit books Rajatarangini, Ramayana and Mahabharata into the
Persian language, the literary language of the Mughal court. (Hence statement 2 is incorrect).

Persian became the language of administration at all levels. The language and culture of Persia
had strong influence on the Mughal rulers. This is also because many Persian artists, craftsmen
and scholars came to India and settled here during the Mughal period. (Hence statement 3 is
correct).

Q.55) Solution (b)

Explanation:

• Babur (1483-1530) was the founder of the Mughal Empire in India and a renowned
Central Asian conqueror. He was born in Fergana, in present-day Uzbekistan, and
descended from both Timur (Tamerlane) and Genghis Khan. Babur had a turbulent early
life, marked by numerous battles and struggles for power.

• In 1526, Babur achieved a significant victory in the Battle of Panipat, where he defeated
Ibrahim Lodi, the Sultan of Delhi, and established his rule over northern India. This
marked the beginning of the Mughal Empire, which would later become one of the most
powerful and influential dynasties in Indian history.

• Babur was not only a skilled military strategist but also a talented poet and diarist. He
wrote his autobiography, titled the "Baburnama," which provides valuable insights into
his life, conquests, and the cultural and social landscape of his time. The Baburnama is
highly regarded as an important historical and literary work. (Hence statement 2 is
correct).

• During his reign, Babur introduced various administrative and military reforms, laying
the foundation for the Mughal Empire's future success. He emphasized the importance
of central authority, effective governance, and maintaining good relations with diverse
communities. Babur was known for his tolerance and respect for different religions, as
demonstrated by his policies towards Hindus and other non-Muslim communities
.
IASBABA’S PRELIMS TEST SERIES (AIPTS) 2024 – TEST 5 (MEDIEVAL HISTORY)

• Although gunpowder was known to India earlier, Babur showed what a skilled
combination of artillery and cavalry could achieve. His victories led to rapid
popularization of gunpowder and artillery in India. (Hence statement 1 is incorrect).

• The control of Kabul and Qandhar strengthened India‟s foreign trade since these two
towns were starting points for caravans meant for China in the east, and the
Mediterranean seaports in the west. Thus, India could take a greater share in the great
trans-Asian trade. (Hence statement 3 is correct).

Q.56) Solution (c)

Explanation:

• Sher Shah Suri (1486-1545), also known as Sher Khan, was a remarkable ruler and
military strategist who briefly interrupted the Mughal rule in India. He established the
Sur Empire, which lasted from 1540 to 1555.

• Sher Shah Suri was born in Sasaram, Bihar, in present-day India. He rose to prominence
as a capable military commander and administrator during the reign of the Mughal
Emperor Humayun. In 1540, he defeated Humayun in the Battle of Kannauj, forcing him
into exile and assuming control over northern India.

• As a ruler, Sher Shah Suri implemented numerous administrative reforms that greatly
influenced the governance of the region. He introduced an efficient revenue system,
known as the "Dahsala," which involved direct measurement and assessment of
agricultural produce for tax collection. He also established a network of roads and rest
houses, famously known as the "Sarais," to facilitate trade and communication
throughout his empire.

• Sher Shah Suri's most notable achievement was the construction of the Grand Trunk
Road, a major thoroughfare stretching over 1,500 miles from Bengal to the Khyber Pass.
This road network played a crucial role in promoting trade, commerce, and cultural
exchange between different regions.

• Sher Shah Suri, introduced several administrative reforms during his reign. He divided
his empire into several provinces, each headed by a governor or a subedar. The
provinces were further divided into fourty seven sarkars. Chief Shiqdar (Law and Order)
and Chief Munsif (Judge) were the two officers in charge of the administration in each
sarkar.
.
IASBABA’S PRELIMS TEST SERIES (AIPTS) 2024 – TEST 5 (MEDIEVAL HISTORY)

• The shiqdar was responsible for maintaining law and order, and ensuring general
administration in his district. He was responsible for the safety and security of the
people, and for maintaining peace and harmony in the district. (Hence option (c) is the
correct answer)

Q.57) Solution (b)

Explanation:

• The peasants who owned the land they tilled were called Khudkasht. They paid land
revenue at customary rates. Some of them had many ploughs and bullocks, which they
let out to their poorer brethren, the tenants or muzarian, who generally paid land
revenue at a higher rate. These two groups were the largest section among the
cultivators in the village. (Hence statement 1 is correct)

• Thus, the village society was highly unequal. The Khudkasht, who claimed to be the
original settlers of the village, often belonged to a single dominant caste or castes.
These castes not only dominated the village society, they exploited the other or weaker
sections. In turn, they were often exploited by the zamindars. (Hence statement 2 is
correct).

Q.58) Solution (a)

Explanation:

Polaj, chachar, and nasaq were terms used in the Mughal revenue administration. Therefore,
the correct matching of the terms and their meanings is:

• Polaj - Land which remained under cultivation every year without being allowed to lie
fallow for a single season. When it remained uncultivated, it was called Parati (fallow).

• Chachar - Land which had been fallow for two to three years and Banjar, if longer than
that. These were assessed at concessional rates; the revenue demand gradually rising till
the full or Polaj rate was paid in the fifth or the eighth year. In this way, the state helped
in bringing virgin and uncultivated wasteland under cultivation.

• Nasaq - A written document that recorded the actual state of cultivation, the amount of
land revenue payable by a peasant, and other details of revenue assessment. It was
widely used in Akbar’s time.
.
IASBABA’S PRELIMS TEST SERIES (AIPTS) 2024 – TEST 5 (MEDIEVAL HISTORY)

Important Points:

• During the Mughal Empire, different types of lands existed, each with its own
characteristics and purposes. Here are some of the main types of lands during that time:

• Khalisa Land: Khalisa lands were directly owned and controlled by the Mughal emperor.
These lands were considered the personal property of the emperor and formed the core
of the imperial domain. The revenue from Khalisa lands directly went to the imperial
treasury.

• Jagir Land: Jagir lands were assigned to nobles, military officers, and other influential
individuals in exchange for their services to the empire. The holders of Jagir lands,
known as Jagirdars, were responsible for collecting revenue from the assigned
territories. They retained a portion of the revenue as their income while forwarding the
rest to the imperial treasury.

• Zamindari Land: Zamindari lands were owned and cultivated by Zamindars, who were
powerful landowners and local administrators. Zamindars held hereditary rights over
these lands and collected revenue directly from the peasants. They were responsible for
managing the land, maintaining law and order, and paying a share of the collected
revenue to the Mughal state.

• Inam Land: Inam lands were granted as non-revenue grants for various purposes. These
lands were often given to religious institutions, scholars, poets, and other individuals as
a reward or support for their contributions to society. The holders of Inam lands were
exempted from paying revenue to the state but were expected to fulfill certain duties or
obligations.

• Crown Land: Crown lands referred to the territories and estates directly controlled by
the Mughal imperial family. These lands were not assigned to Jagirdars or Zamindars but
were managed by officials appointed by the emperor. The revenue from Crown lands
directly benefited the imperial family

Q.59) Solution (d)

Explanation:

• The real founder of the Muzaffarid dynasty of Gujarat was Ahmad Shah I (1411–43), the
grandson of Muzaffar Shah. (Hence statement 1 is correct).
.
IASBABA’S PRELIMS TEST SERIES (AIPTS) 2024 – TEST 5 (MEDIEVAL HISTORY)

• During his long reign, he brought the nobility under control, settled the administration,
and expanded and consolidated the kingdom. He shifted the capital from Patan to the
new city of Ahmedabad, the foundation of which he laid in 1413. (Hence statement 2 is
correct).

• He was a great builder and beautified the town with many magnificent palaces and
bazars, mosques and Madarsas.

• He drew on the rich architectural traditions of the Jains of Gujarat to devise a style of
building which was markedly different from Delhi. Some of its features are slender
turrets, exquisite stone-carving and highly ornate brackets. (Hence statement 3 is
correct).

• The Jama Masjid in Ahmedabad and the Tin Darwaza are fine examples of the style of
architecture during his time.

• He imposed Jizyah on the Hindus in Gujarat, which had not been imposed on them
earlier. All these have led many medieval historians to hail Ahmad Shah as a great
enemy of the infidels, while many modern historians have called him a bigot. The truth,
however, appears to be more complex. (Hence statement 4 is correct).

• While Ahmad Shah acted as a bigot in ordering the destruction of the Hindu temples, he
did not hesitate to induct the Hindus in his government. Manik Chand and Moti Chand,
belonging to the Bania or the trader community, were ministers under him.

Q.60) Solution (c)

Explanation:

• Abul Fazl wrote a book, Akbarnama (the life of Akbar) of which is a section of Ain-i-
Akbari. This section deals with the laws and the revenue system of the empire, and also
describes the condition of the country. (Hence statement 1 is correct).

• The Muslim teachers were the first to be invited to the IbadatKhana. Later others were
invited to hold discussion with the king- Hindus, Parsis, Jains, and Christians. The
portugese governor sent Missionaries, in the hope that they might be able to convert
the emperor to Christianity. After all these discussions Akbar decided that he had found
the way. He suggested a new religious path. This was based on the common truths of all
religions and a few rules taken from various religions. (Hence statement 2 is correct).
.
IASBABA’S PRELIMS TEST SERIES (AIPTS) 2024 – TEST 5 (MEDIEVAL HISTORY)

• This religious path was later referred to as the Din-i-ilahi or divine monotheism- the
Religion of the one God. (Hence statement 3 is correct).

Important points about Din-i-ilahi:

• Syncretic Religion: Din-i-Ilahi was a syncretic religious movement founded by Emperor


Akbar in the late 16th century.

• Purpose: Akbar aimed to create a common moral and ethical framework that would
bridge the gap between different religious communities in his empire.

• Divine Faith: The term "Din-i-Ilahi" translates to "Religion of God" in Persian.

• Principles: Din-i-Ilahi incorporated elements from various religions, including Islamic


monotheism, Hinduism, Zoroastrianism, Jainism, and Christianity.

• Unity of God: The movement emphasized the concept of divine unity, focusing on the
belief in a single God.

• Debates and Discussions: Akbar invited scholars, theologians, and religious leaders from
different faiths to engage in discussions and debates at his court to explore
commonalities among religions.

• Inclusivity: Din-i-Ilahi aimed to create a universal religion that transcended the


boundaries of existing belief systems.

• Limited Impact: The movement remained largely confined to the Mughal court and did
not gain widespread popularity among the general population.

• Criticism: Din-i-Ilahi faced criticism and skepticism from Muslims and Hindus who
viewed it as an attempt to create a new religion and deviate from established traditions.

• Decline: After Akbar's death, subsequent Mughal emperors did not actively promote or
follow Din-i-Ilahi, leading to its gradual decline and eventual disappearance.

Q.61) Solution (d)

Explanation:

• The Battle of Haldighati was fought in 1576 CE between the forces of Maharana Pratap,
the Rajput ruler of Mewar, and the Mughal army led by Raja Man Singh of Amber.
.
IASBABA’S PRELIMS TEST SERIES (AIPTS) 2024 – TEST 5 (MEDIEVAL HISTORY)

• The Battle of Haldighati was not a struggle between the Hindus and the Muslims. It
was fought between the forces of Maharana Pratap, who was a Hindu king and a Rajput
warrior, and the Mughal army led by their general, Man Singh. Maharana Pratap and his
forces were also primarily Hindu, but the conflict was not based on religious grounds.
(Hence statement 1 is incorrect)

• The Battle of Haldighati was fought between Maharana Pratap's forces and the Mughal
army, but it was not led by Bairam Khan. Bairam Khan was a trusted advisor of Akbar
and was appointed as the regent after Akbar's accession to the throne. However, by the
time of the Battle of Haldighati, he had been replaced by Man Singh, who led the
Mughal forces. (Hence statement 2 is correct).

Q.62) Solution (c)

Explanation:

• With the arrival of Turks in India during the tenth century, a new language, Persian, was
introduced in the country. (Hence statement 1 is correct).

• Firdausi, also known as Ferdowsi or Hakim Abu'l-Qasim Firdawsi Tusi, was a famous
Persian poet at the court of Mahmud Ghazni. He was born in Tus, Iran, during the
Samanid dynasty. He has been described as ‘the immortal Homer of the East’. Firdausi
is renowned for his epic poem, the Shahnama ("Book of Kings"), which is considered the
national epic of Iran and one of the greatest works of Persian literature. (Hence
statement 2 is incorrect).

• In course of time, Persian became not only the language of administration and
diplomacy, but also the language of the upper classes and their dependents. (Hence
statement 3 is correct).

• Tuti Nama, a Persian text written by Zia al-Din Nakhshabi, was a redaction of a 12th
century Sanskrit anthology titled Sukasaptati or Seventy Tales of the Parrot. It is a
collection of interconnected stories and fables framed within a narrative structure. The
central story of Tutinama revolves around a parrot that possesses the ability to speak
and understand human language. The parrot becomes the pet of a prince and begins to
narrate stories to him. Each story serves as a moral lesson or a source of entertainment.
(Hence statement 4 is correct).
.
IASBABA’S PRELIMS TEST SERIES (AIPTS) 2024 – TEST 5 (MEDIEVAL HISTORY)

Q.63) Solution (c)

Explanation:

The systems of Watans and Saranjami are associated with Marathas Kingdom. Under
Saranjami system, lands are granted (initially nonhereditary) for maintenance of troops for
military service, found among the Maratha community in Maharashtra. The land was mostly in
the form of a rural Watan (rights given in reward for previous service or merit) or Jagir, its
owner being entitled to extract revenue from the villages included in the territory. (Hence
option (c) is correct).

Q.64) Solution (a)

Explanation:

• Sufism is a mystical form of Islam that emphasizes on the spiritual connection with God
and the pursuit of inner purification. It became popular in India during the medieval
period and had a significant influence on the religious and cultural life of the country.

• Moinuddin Chishti was a prominent Sufi saint who came to India in the 12th century
and settled in Ajmer. However, he was not the first Sufi saint to come to India and
settle. Sufi saints had been coming to India since the 8th century, and many of them had
settled in different parts of the country. (Hence statement 1 is incorrect).

• Ziyarat is a term used for the pilgrimage to the tombs of Sufi saints. It is a significant
aspect of Sufi practice and reflects the belief that the blessings of the saints continue
even after their physical death. (Hence statement 2 is correct).

• Maktubat refers to compilations of letters written by the Sufi masters, addressed to


their disciples and associates. These letters often contain spiritual guidance and insights
into the Sufi way of life. (Hence statement 3 is incorrect).

• Malfuzat is a collection of discourses, conversations, and teachings of the Sufi saints. It


includes their sayings, anecdotes, and stories that illustrate their teachings and provide
guidance on the spiritual path. (Hence statement 4 is incorrect).

Q.65) Solution (c)

Explanation:
.
IASBABA’S PRELIMS TEST SERIES (AIPTS) 2024 – TEST 5 (MEDIEVAL HISTORY)

• Vallabhacharya was a prominent Hindu philosopher and teacher who lived in the 16th
century in North India. He was a devotee of Lord Krishna and is known for his
contributions to the Bhakti movement. (Hence statement 1 is correct).

• Vallabhacharya's philosophy is known as Shuddha Advaita or pure non-dualism.


According to this philosophy, the individual soul (Jivatma) is not separate from the
ultimate reality (Paramatma), and the goal of life is to attain union with the divine.

• Vallabhacharya also propounded the Pustimarga or the path of grace, which emphasizes
the importance of devotion to Lord Krishna and the worship of the divine in the form of
the "Lila" or the divine play. (Hence statement 2 is correct).

• Vallabhacharya was instrumental in establishing the Srinathji Temple at Nathdwara in


Rajasthan, which is one of the most important pilgrimage sites for the followers of
Pushtimarga. The temple is dedicated to Lord Krishna and is known for its beautiful idol
of the deity, which is believed to have been brought from Mathura by Vallabhacharya
himself. (Hence statement 3 is correct).

Q.66) Solution (d)

Explanation:

• Dadu Dayal was a 16th century poet and saint from Rajasthan, and is associated with
the Dadu Panth, a religious tradition that emphasizes devotion to God and the pursuit of
spiritual knowledge. (Hence pair 1 incorrectly matched).

• Ravidas was a 15th century saint and poet from Uttar Pradesh, and is associated with
the Bhakti movement. He is venerated by both Hindus and Sikhs, and is known for his
teachings on social equality and the unity of all people. (Hence pair 2 incorrectly
matched).

• Bhagat Pipa was a 14th century saint and poet from Rajasthan, and is associated with
the Nirguna Bhakti movement, which emphasizes devotion to God without any
attributes or qualities. (Hence pair 3 incorrectly matched).

• Therefore, the correctly matched pairs would be:

o Dadu Dayal: Rajasthan

o Ravi Das: Uttar Pradesh


.
IASBABA’S PRELIMS TEST SERIES (AIPTS) 2024 – TEST 5 (MEDIEVAL HISTORY)

o Bhagat Pipa: Rajasthan

Q.67) Solution (d)

Explanation:

• Ramanuja Acharya, also known as Ramanujacharya, was a prominent philosopher,


theologian, and spiritual leader who lived in South India during the 11th and 12th
centuries. He was a key figure in the Bhakti movement and made significant
contributions to the development of Vaishnavism, particularly in the Sri Vaishnava
tradition.

• His philosophy is called Visishtadvaita, which means "qualified non-dualism". It posits


that the ultimate reality (Brahman) is both one and many, and that the individual souls
(atman) are distinct from Brahman but also dependent on it. (Hence statement 1 is
correct).

• Sri Ramanujacharya underwent philosophical training in the Sankara School of thought.


Ramanuja was a teacher-reformer and a great organizer. He challenged the monist
ideology of Adi Shankara, who argued for the identity of Brahman and atman.

• Ramanujacharya believed in the ultimate reality of Brahman, but also in the reality of
the individual soul and the material world. His philosophy of Visishtadvaita influenced
many thinkers and developed into a separate tradition. (Hence statement 2 is correct).

• He influenced some temple authorities to permit the social groups outside the
Varnashram system (such as Dalits and women) to enter into the temple at least once a
year, breaking the Brahmanical tradition of temple entry restricted to only upper-caste
men. This practice, known as "temple-entry movement", was a major social reform in
South India. (Hence statement 3 is correct).

Q.68) Solution (a)

Explanation:

• Kabir Daswas born in a poor family considered to be weavers. He was initiated into
bhakti by Guru Ramananda. He was present in the 14th and 15th century. He is a poet
saint and many of his compositions are found in Adigranth Sahib. One of the striking
.
IASBABA’S PRELIMS TEST SERIES (AIPTS) 2024 – TEST 5 (MEDIEVAL HISTORY)

features of his compositions was that they had traditions followed by Islam, Vedic as
well as yogic traditions.

• Kabir Das, often referred to simply as Kabir, was a 15th-century Indian poet,
philosopher, and mystic. He is revered as one of the most influential figures in Indian
literature and spiritual traditions. Kabir's writings and teachings embody a unique
synthesis of Hindu and Islamic philosophies, emphasizing the unity of God and the
inherent divinity within every individual.

• Born into a Muslim family, Kabir later embraced a form of spirituality that transcended
religious boundaries. His poetry, written in the vernacular language of Hindi, known as
"Bhakti poetry," resonates with people from various religious backgrounds. Kabir's
verses are marked by simplicity, directness, and a strong social and moral critique of
religious rituals and caste-based discrimination.

• Kabir's teachings revolve around the pursuit of spiritual realization through love,
devotion, and the inner experience of God. He rejected dogmas and rituals, emphasizing
the importance of personal experience over external forms of worship. His teachings
stressed the idea that the Divine can be realized directly within one's own heart,
without the need for intermediaries or external authorities.

• Kabir's poetry explores themes such as the illusory nature of the material world, the
power of love and devotion, and the unity of all beings. His verses often use metaphors
and vivid imagery drawn from everyday life to convey deep spiritual truths.

• Despite facing criticism and opposition during his lifetime, Kabir's teachings spread far
and wide, inspiring countless followers and influencing subsequent generations of poets
and spiritual seekers. His verses continue to be sung and recited in devotional
gatherings and have been compiled in various collections, such as the "Bijak" and "Kabir
Granthavali."

• Kabir's legacy as a social reformer, spiritual guide, and poet remains significant,
transcending religious and cultural boundaries. His teachings embody a message of
unity, love, and inner realization that continues to resonate with people seeking a
deeper understanding of spirituality and the human condition.

Q.69) Solution (d)

Explanation:
.
IASBABA’S PRELIMS TEST SERIES (AIPTS) 2024 – TEST 5 (MEDIEVAL HISTORY)

The wave of Bhakti movement started from south India, by Alvaras-the devotees of Lord Vishnu
and Nayanars-devotees of Shiva. They travelled to various places singing hymns in Tamil in
praise of their gods. Later, many temples were built that became the sacred places for
pilgrimage. The compositions by the poet saints became part of temple rituals in these shrines.
Some historians also believed that the Altars and Nayanars initiated a movement of protest
against the caste system and the dominance of Brahmanas or at least attempted to reform the
system. Influences made by the Bhakti movement on the society:

Religious effects

• Reformed Hinduism

• Lowered the prestige of the Brahmins

• Checked the propagation of Islam

• Emergence of Sikhism

• Setback to Buddhism

Social impact

• Improvement in the social relation between the Hindus and Muslims (Hence statement
1 is correct).

• The better social status of lower castes. (Hence statement 2 is correct).

• Promotion of social service

• Development of composite art in the society. (Hence statement 4 is correct).

• Enrichment of literature. (Hence statement 3 is correct).

Q.70) Solution (c)

Explanation:

• Samkhya: according to this school, a person can attain salvation through the
acquisition of real knowledge and participation in the life cycle can be ended forever.
This knowledge can be acquired through pratyaksha (perception), anumanya
(inference) and (hearing).
.
IASBABA’S PRELIMS TEST SERIES (AIPTS) 2024 – TEST 5 (MEDIEVAL HISTORY)

• Nyaya: This school advocates the theory of pramanas (proof), which means of obtaining
true knowledge. According to it, the pradhanpramana or principal means of obtaining
knowledge is pratyakshapramana (knowledge obtained through the five senses). There
are also other pramanas like anumana (inference, through which we can obtain true
knowledge) and shabdapramana (a statement of an expert). (Hence statement 1 is
correct).

• Vaisheshika: This school propounded by Kanada Kashyapa gives importance to


discussion of dravya or material elements. This school propounded the atomic theory
and can be marked as the beginning of the physics in ancient India. Earth, water, fire, air
and ether (sky) are the aggregates, which combined give rise to new objects. But the
scientific view was diluted with belief in God and spiritualism, and this school put its
faith in both heaven and salvation. (Hence statement 2 is incorrect).

Q.71) Solution (b)

Explanation:

• The term which Indo-Persian sources of the Mughal period most frequently used to
denote a peasant was raiyat (plural, riaya) or muzarian. In addition, we also encounter
the terms kisan or asami (Hence statement 1 is correct).

• Sources of the seventeenth century refer to two kinds of peasants – Khud- Kashta and
Pahi-Kashta. The former were the residents of the village in which they held their
lands.

• The latter were non-resident cultivators, who belonged to some other village, but
cultivated lands elsewhere on a contractual basis. People became Pahi-Kashta either out
of choice – for example, when terms of revenue in a distant village were more
favourable – or out of compulsion – for example, forced by economic distress after a
famine. (Hence statement 2 is incorrect).

• Peasant lands during the Mughal times were not generally bought and sold in the same
way as private property. Instead, the Mughal state recognized the rights of peasant
cultivators to hold and work the land, and collected taxes from them in return. These
cultivators were known as raiyats or zamindars, and their rights to the land were
typically hereditary and transferable within the family. (Hence statement 3 is correct)
.
IASBABA’S PRELIMS TEST SERIES (AIPTS) 2024 – TEST 5 (MEDIEVAL HISTORY)

Q.72) Solution (c)

Explanation:

• The term Jins-i Kamil literally means perfect crops. It referred to cash crops. Since they
brought income to the country, Mughal rulers encouraged farmers of cash crops.
(Hence statement 1 is correct)

• The most important jins-i-kamil was cotton and sugar cane. Cotton was cultivated in
Central India and the highlands of Deccan. Bengal was famous for sugar cane. Pulses and
oil seeds like mustard also were considered cash crops. It shows there was mixture of
food crops and cash crops in the country.

• Tobacco, which arrived first in the Deccan, spread to northern India in the early years of
the seventeenth century. Akbar and his nobles came across tobacco for the first time in
1604. At this time smoking tobacco (in hookahs or chillums) seems to have caught on in
a big way.

• Jahangir was so concerned about its addiction that he banned it. This ban was totally
ineffective because by the end of the seventeenth century, tobacco had become a major
article of consumption, cultivation and trade all over India. (Hence statement 2 is
correct)

• Bengal was a major center for sugar production during the Mughal period, and its sugar
was exported to other parts of India and beyond. (Hence statement 3 is correct)

Q.73) Solution (a)

Explanation:

• The Humayunnama is a historical account of the reign of the Mughal Emperor Humayun,
written by his court historian, Gulbadan Begum, in Persian. The Humayunnama provides
a detailed account of Humayun's life and reign, including his battles, military campaigns,
and personal life. It also describes the political and social conditions of the time, as well
as the culture and customs of the Mughal court. (Hence statement 1 is incorrectly
matched)

• The ‘Padshahnama’ is a historical chronicle of the reign of the Mughal Emperor Shah
Jahan, written by his court historian, Abdul Hamid Lahori. It is also known as
‘Padshahnama’ or "Chronicle of the Emperor". The Badshahnama covers Shah Jahan's
.
IASBABA’S PRELIMS TEST SERIES (AIPTS) 2024 – TEST 5 (MEDIEVAL HISTORY)

reign from 1627 to 1658 and provides a detailed account of his military campaigns,
administrative policies, building projects, and personal life. (Hence statement 2 is
incorrectly matched)

• The ‘Alamgirnama’ is a historical chronicle of the reign of the Mughal Emperor


Aurangzeb, written by his court historian, Muhammad Kazim. It covers Aurangzeb's
reign from 1658 to 1707 and provides a detailed account of his military campaigns,
administrative policies, religious views, and personal life (Hence statement 3 is correctly
matched).

Q.74) Solution (c)

Explanation:

• Mahzarnama or the Infallibility Decree was issued by Akbar in 1579. It declared that
the king was the final authority in matters of religion. This decree granted the freedom
of religion to all, including Hindus, and established the principle of Sulh-i-Kul, which
.
IASBABA’S PRELIMS TEST SERIES (AIPTS) 2024 – TEST 5 (MEDIEVAL HISTORY)

means universal peace. It was not issued by Aurangzeb. (Hence statement 1 is


incorrect).

• The decree was drafted by two of Akbar's close associates, Sheikh Mubarak and Abul
Fazl. (Hence statement 2 is correct)

• Under the decree, Akbar declared himself as the final authority in religious matters and
became the final interpreter of the Quran, superseding the ulema or the religious
scholars. The Mahzar was a significant step in Akbar's efforts to create a syncretic
religion that combined elements of various religions prevalent in India at that time. The
decree allowed Akbar to promote religious tolerance and bring people of different faiths
together. (Hence statement 3 is correct).

• The Mahzarnama was considered a landmark in the religious policy of Akbar and was an
attempt to bring religious harmony among his subjects.

Q.75) Solution (a)

Explanation:

• The Dahsala system was a system of revenue collection introduced by Todar Mal, a
finance minister in the court of Akbar. (Hence statement 1 is correct).

• It was based on the assessment of the quality and productivity of the land and the crops
grown on it. It was not an annual assessment but a permanent settlement, which means
that the revenue rates were fixed for a longer period, which is for 10 years. (Hence
statement 2 is incorrect).

• The system was only used in the Empire's major provinces, which included Multan,
Delhi, Allahabad, Awadh, Agra, and Lahore. However, at no point of time, all the land in
a particular region was measured. That would suggest that even in measured territories
some territories remained unmeasured. In such a situation even in the Zabti regions,
other methods of assessment and collection were followed in almost all parts of the
country. (Hence statement 3 is incorrect).

Important Points/Value Add:

• Revenue Administration: The Dahsala system was a revenue administration system that
brought uniformity and efficiency to the collection of revenue throughout the empire. It
aimed to streamline the revenue assessment and collection process.
.
IASBABA’S PRELIMS TEST SERIES (AIPTS) 2024 – TEST 5 (MEDIEVAL HISTORY)

• Land Measurement and Assessment: Raja Todar Mal implemented a standardized


system of land measurement, known as the "bigha," which helped in accurate
assessment and taxation of land. This system helped in preventing discrepancies and
disputes related to land measurement.

• Fixed Revenue Share: The Dahsala system introduced a fixed revenue share, usually
one-third or one-fourth of the produce, to be collected as revenue from the cultivators.
This provided stability and predictability to the revenue collection process.

• Integration of Local Practices: Raja Todar Mal incorporated local systems and practices
into the Dahsala system, allowing for some flexibility and accommodation of regional
variations. This helped in gaining acceptance and cooperation from the local population.

• Land Record-keeping: The Dahsala system emphasized the maintenance of accurate and
detailed land records. This allowed for better land administration, reduced disputes, and
facilitated future planning and development.

Q.76) Solution (d)

Explanation:

Abdul Razak was a Persian pilgrim who visited Vijayanagara during the reign of Deva Raya II. He
was impressed by the city and described it as a unique and well-defended city surrounded by a
seven-round baladya fort. He also mentioned that the city had many markets, parks, wide
streets, and buildings. According to Razak, the people of Vijayanagara loved rose and jasmine
flowers, and precious gems, diamonds, and turquoise were sold on the streets without any fear.
He also noted that the king had appointed many soldiers to protect the city and the palace, and
that the throne was adorned with pearls. The palace was surrounded by gardens and fountains,
and the Navratri festival was celebrated with great pomp.

Q.77) Solution (b)

Explanation:

• Hampi was the capital city of the Vijayanagara kingdom, serving as the capital of the
Vijayanagara Empire in the 14th century. The city is located in northern Karnataka, India
and is famous as a pilgrimage center for both the Hindu and Jain religions. (Hence
statement 1 is correct).
.
IASBABA’S PRELIMS TEST SERIES (AIPTS) 2024 – TEST 5 (MEDIEVAL HISTORY)

• It is situated on the banks of the Tungabhadra River. In its heyday, Hampi-Vijayanagara


was considered the world's second-largest medieval-era city after Beijing by the year
1500 CE. (Hence statement 2 is incorrect).

• The Group of Monuments at Hampi was added to the UNESCO World Heritage Sites in
1986, and the site has about 1,600 monuments that include temples, palaces,
marketplaces, and public buildings that offer a glimpse into the grandeur of the
Vijayanagara Empire. (Hence statement 3 is correct).

Q.78) Solution (b)

Explanation:

• The Vijayanagara Empire, which existed from the 14th to the 17th century in South
India, had a diverse range of occupations and economic activities that contributed to the
empire's prosperity.

• The primary occupation of the people in the Vijayanagar Empire was agriculture, and
the rulers of the kingdom played a significant role in promoting its growth by facilitating
the development of irrigation facilities. (Hence statement 1 is incorrect).

• They constructed new tanks and dams across the rivers, such as the Tungabhadra, and
excavated canals, as noted by Nuniz.

• Additionally, there were several seaports along the Malabar coast, including the main
port of Cannanore. The kingdom enjoyed thriving trade relationships with various
countries, including Arabia, Persia, South Africa, Portugal, Burma, the Malay peninsula,
and China.

• The primary exports were cotton and silk clothing, spices, rice, iron, saltpetre, and
sugar, while the main imports included horses, pearls, copper, coral, mercury, China
silk, and velvet clothing. The art of shipbuilding had also progressed significantly during
this time. (Hence statements 2 and 3 are correct).

Q.79) Solution (d)

Explanation:
.
IASBABA’S PRELIMS TEST SERIES (AIPTS) 2024 – TEST 5 (MEDIEVAL HISTORY)

• The Raichur doab, situated between the rivers Krishna and Tungabhadra, was an area of
great significance as it was fertile and abundant in mineral resources. The eastern part
of this region was home to the famous diamond mines of Golconda. (Hence statement 2
is correct).

• Both the Bahmani Sultanate and the Vijayanagara Empire had the potential to expand
their territories only across the Tungabhadra in the Deccan due to their geographic
locations. The conflicts between the two powers were inconclusive, and the status quo
was maintained with neither side gaining a clear advantage.

• In 1504 CE, the Bahmanis regained control of the Raichur doab, but their hold on the
area was short-lived. With the rise of Krishna Deva Raya, the Bahmanis lost their hold on
several towns including Raichur, Mudkal, and Nalgonda.

• It is worth noting that the Raichur doab had been a disputed territory even in earlier
periods, with conflicts between the western Chalukyas and the Cholas in the past and
between the Yadavas and the Hoysalas in later times.

• Many people tend to view the conflicts between the Vijayanagara Empire and the
Bahmani Sultanate as a clash between Hindus and Muslims. However, the historical
facts suggest that these battles were not fought over religious differences but rather
for territorial and economic gain. (Hence statement 1 is incorrect).

• The consequences of these military confrontations were catastrophic, with widespread


destruction in the contested areas and the surrounding regions, and a significant loss of
both human life and property.

Q.80) Solution (d)

Explanation:

• Alauddin Bahman Shah, also known as Hasan Gangu, established the Bahmani kingdom
in 1347 AD. He was succeeded by his son Muhammad Shah I.

• Firoz Shah Bahmani, who ruled from 1397 to 1422, was one of the most remarkable
rulers of the Bahmani kingdom. He built an observatory near Daulatabad and inducted
Hindus into the administration on a large scale.

• Ahmad Shah, who ruled from 1422 to 1436, was known as Wali due to his association
with the Sufi saint Gesu Daraz. He shifted his capital from Gulbarga to Bidar.
.
IASBABA’S PRELIMS TEST SERIES (AIPTS) 2024 – TEST 5 (MEDIEVAL HISTORY)

• Mahmud Gawan served as the prime minister of Muhammad Shah Bahmani II from
1463 to 1482 and was bestowed with the title of Malik-ul-Tujjar. He established multiple
Madrasas in Bidar to promote Islamic education. Mahmud Gawan's tenure saw the
Bahmani Sultanate rise to become the most dominant power in southern India.
However, he engaged in a series of conflicts with Mahmud Khalji of Malwa over Berar, a
prosperous region renowned for cotton production.

• (Hence the correct answer is option d).

Q.81) Solution (c)

Explanation:

• Muhammad Gawan, an advocate of education and literature, established 'Madrasa-e-


Mahmood Gawan' in Bidar, where he completed the exceptional madrasa building in
1472 AD and began delivering lectures on various subjects, including philosophy,
geography, mathematics, and language. Furthermore, he generously offered free
lodging and board to the college's students and faculty members. The Madrasa library
housed approximately 3,000 books, and Gawan, a scholar himself, made significant
contributions to the advancement of education.

• In addition to being an education patron, Mohammad Gawan was also a literary


enthusiast. He invited foreign scholars to his kingdom and, being a literary figure
himself, made significant contributions to the Saraswat community by creating "Diwan-
e-Ashra" and "Raujat-ul-Inna."

Q.82) Solution (a)

Explanation:

Ministers of Bahmani Sultans dynasty

To ensure efficient governance like the Delhi Sultanate, Bahman Shah divided his kingdom into
four geographical divisions named tarafs, with a governor appointed for each. These governors
were in charge of their respective provinces' military, administration, and revenue collection.
The four tarafs were Gulbarga, Daulatabad, Bidar, and Berar.

Mohammed I appointed a council of eight ministers of state:


.
IASBABA’S PRELIMS TEST SERIES (AIPTS) 2024 – TEST 5 (MEDIEVAL HISTORY)

• Vakil-us-saltana or lieutenant of the kingdom, the immediate subordinate of the


sovereign.

• Waziri-kull, who supervised the work of all other ministers;

• Amir-i-jumla, minister of finance; (Hence pair 1 is correctly matched).

• Wasir-i-ashraf, minister of foreign affairs and master of ceremonies; (Hence pair 2 is


incorrectly matched).

• Nazir, assistant minister for finance;

• Peshwa who was associated with the lieutenant of the kingdom;

• Kotwal or chief of police and city magistrate in the capital, and

• Sadr-i-jahan or chief justice and minister of religious affairs and endowments. (Hence
pair 3 is incorrectly matched).

Q.83) Solution (c)

Explanation:

• Nandithimmana: He was very close to Sri Krishna Devaraya and became the court poet
of Vijayanagara. His major works are 'Vanivilasam' and 'Parijatapaharanam'.
'Parijatapaharanam contains details of Krishnadevaraya's life achievements.

• Tenali Ramakrishna: He belongs to Guntur district. He had Tenali Annayya. Became


close to Krishna Deva Raya due to his sense of humour. He has written works on serious
subjects as well as humor. He composed Panduranga Mahatma, Ghatikachal Mahatre',
'Udbhataradhya Charitam'. He was said to have mastered Telugu, Kannada, Tamil and
Marathi languages and was bitten by a snake.

• Doorzati: Belonging to Sri Kalahasti village, Sri Kalahastiswara Mahatma wrote the
works 'Sri Kalahastiswara Shatakam'.

• Madayagari Mallanna: He belongs to the Royal area. Rajasekhara Charitam is his main
work. He used to go to war land with the king.

• Ayyalaraj Ramabhadra: Belongs to Kadapa district. His main works are


'Ramabhyadayam', 'Sakala Kathasara Sankaram'.
.
IASBABA’S PRELIMS TEST SERIES (AIPTS) 2024 – TEST 5 (MEDIEVAL HISTORY)

• Pingli Suranna: He is said to be from Pingali village of Krishna district. His main works
are Garuda Puranam, 'Prabhavati Pradyumnamu, Raghava Pandaveiyamu',
'Kalapurnodayam'.

• Ramaraja Bhushan: His hometown is Nallur, a disciple of Allasani Peddanna, a famous


musician who can play veena and other instruments, also a writer; his main works are
'Kavyalankara Parshamu', 'Harishchandra Nalopakhyayanam' and 'Narasabhupaliyam'.

Q.84) Solution (d)

Explanation:

• Nicolo Conti, an Italian, was at Vijayanagara in about 1420, just after the accession of
Devaraya I. The first known foreign traveller, he mentions the fortifications of the city
and the thousands of men employed in the army of the rulers.

• Athanasius Nikitin was the first documented Russian traveller who visited India. He
described the conditions of Vijayanagar Empire and Bahmani kingdom under
Muhammad III in his Voyage to India.

• Abdur Razzaq (Persian) visited the Vijayanagar Kingdom at the time of Dev Raya II as an
ambassador of Shah Rukh, the Timurid dynasty ruler of Persia. Athanasius Nikitin was
the first documented Russian traveller who visited India.

• Domingo Paes (sometimes spelt Pais) was a traveller from Portuguese India, who visited
the Vijayanagara Empire, located on the southern portion of the Deccan Plateau in
around 1520.

Do You Know?

• Duarte Barbosa, a Portuguese explorer and writer, was one of the foreign travelers who
visited the Vijayanagara Empire during the 16th century. Barbosa marveled at the
magnificence and grandeur of Vijayanagara, which he described as "the best provided
city of the world." He was impressed by the city's vast size, architectural splendor, and
the impressive structures, including temples, palaces, and markets.

Q.85) Solution (b)

Explanation:
.
IASBABA’S PRELIMS TEST SERIES (AIPTS) 2024 – TEST 5 (MEDIEVAL HISTORY)

• During the reign of Krishna Deva, the taxation system in the Vijayanagara Empire was
primarily based on the land and industry taxes. The tax rate on land was determined
based on the quality of the land. The fertile and well-irrigated lands had higher taxes
compared to less fertile lands. This was a common practice in many medieval kingdoms
and empires, as the land was considered one of the most valuable resources for
agriculture and revenue. (Hence statement 1 is correct).

• In addition to land tax, the private owners of workshops and industries also had to pay
an industry tax. This tax was levied on the businesses that produced goods or services in
the empire. The tax rate was calculated based on the nature and size of the industry.
For instance, a small workshop may have to pay a lower tax than a large-scale industry.
(Hence statement 2 is correct).

• Overall, the taxation system of Krishna Deva's reign was designed to generate revenue
for the empire while ensuring a fair distribution of the tax burden among different
segments of society. The taxes collected were used for various purposes such as building
infrastructure, maintaining law and order, and financing the military.

Q.86) Solution (b)

Explanation:

• Sufism is a mystical and spiritual Islamic belief system that emphasizes the inner search
for God and a direct personal experience of the divine. It is sometimes referred to as
Islamic mysticism or Islamic spirituality. (Hence statement 1 is incorrect).

• Sufism originated in the early centuries of Islam and has been practiced by Muslims in
various parts of the world. The followers of Sufism are called Sufis, and they believe in
seeking a closer and more intimate relationship with God through various spiritual
practices, such as meditation, prayer, recitation of the Quran, and other forms of
worship.

• Sufism also emphasizes the importance of love, compassion, and the pursuit of inner
peace, as well as the cultivation of virtues such as humility, generosity, and kindness.
Sufis often express their spiritual experiences through poetry, music, and dance, and
some of the most famous poets and musicians in the Islamic world have been Sufis.
(Hence statement 2 is correct).
.
IASBABA’S PRELIMS TEST SERIES (AIPTS) 2024 – TEST 5 (MEDIEVAL HISTORY)

• Sufism has had a significant influence on Islamic culture, particularly in the fields of
art, literature, and music. Many Sufi orders or brotherhoods have been established over
the centuries, each with its own distinctive practices and teachings. Sufism has also
been a subject of study and interest for scholars and non-Muslims who are interested in
the mystical aspects of Islam. (Hence statement 3 is correct).

Q.87) Solution (c)

Explanation:

The title "Mahboobillah" is a reverential designation bestowed upon Shaik Nizam-ud-din


Auliya, a revered Sufi saint who resided in Delhi, India during the 16th century. He held a
significant position within the Chishti Sufi order and garnered immense admiration within Sufi
communities globally. "Mahboobillah" symbolizes "Beloved of Allah" and serves as a profound
token of reverence and acknowledgment for his elevated spiritual status and unwavering
devotion to God.

More about Shaik Nizam-ud-din Auliya:

• Shaik Nizam-ud-din Auliya was a prominent Sufi saint and spiritual leader in medieval
India.

• He lived during the 13th and 14th centuries and played a significant role in spreading
the Chishti Sufi order in the Delhi Sultanate.

• Hazrat Nizamuddin Auliya emphasized love, compassion, and devotion to God in his
teachings.

• He sought to connect with people of all backgrounds, irrespective of social status or


religious affiliations.

• Nizamuddin Auliya's approach to spirituality emphasized the unity of religions and the
belief in divine love as the path to enlightenment.

• He established a hospice called the Khanqah in Delhi, which became a center for
spiritual gatherings and discussions.

• Hazrat Nizamuddin Auliya had a large following of disciples, both Muslim and non-
Muslim, who were attracted to his spiritual wisdom and humanitarian values.
.
IASBABA’S PRELIMS TEST SERIES (AIPTS) 2024 – TEST 5 (MEDIEVAL HISTORY)

• He was known for his empathy towards the poor and marginalized and was generous in
distributing food and other essentials to the needy.

• His teachings emphasized social welfare and service to humanity.

• Nizamuddin Auliya's tomb, known as the Nizamuddin Dargah, located in Delhi, is a


revered pilgrimage site and continues to inspire spiritual seekers.

• His teachings are cherished for their profound spiritual wisdom and universal message
of love and compassion.

Q.88) Solution (c)

Explanation:

• Sufis were organized into 12 orders or Silsilahs. (Hence statement 1 is correct).

• Buddhism was a prevalent religion in Central Asia before Islam arrived, and the story of
Buddha as a virtuous man had become a part of Islamic folklore. Despite the
introduction of Islam, yogis continued to travel to West Asia, and the Persian translation
of the Sanskrit yogic text Amrit-Kund was still available. Translated into Persian and
Arabic as Hauz-al-Hayat or Cistern of the Water of Life. Amritakunda became well-
known, especially in Sufi circles. (Hence statement 2 is correct).

• The bond between the instructor, also known as pir, and his students, or murid, was
essential in the Sufi system. (Hence statement 3 is correct).

• Sufism, also known as Islamic mysticism, is a spiritual tradition within Islam that seeks to
attain a deeper understanding of God, achieve spiritual closeness, and strive for inner
purification. Sufis, known as "Sufi mystics" or "Dervishes," engage in practices and
disciplines aimed at purifying the heart, purging the ego, and attaining spiritual
enlightenment.

• Central to Sufism is the concept of "Tasawwuf," which refers to the path of spiritual
realization and transformation. Sufis follow a spiritual guide, known as a "Sheikh" or
"Murshid," who provides guidance, mentoring, and teachings on the path to spiritual
growth.

• Sufis employ various practices and methods to deepen their connection with the divine.
These include meditation, remembrance of God (Zikr), recitation of sacred texts and
poetry, music and chanting (Sama), and engaging in acts of service and charity. Through
.
IASBABA’S PRELIMS TEST SERIES (AIPTS) 2024 – TEST 5 (MEDIEVAL HISTORY)

these practices, Sufis seek to transcend the material world and experience a direct,
intimate connection with God.

Q.89) Solution (b)

Explanation:

• During the reign of Krishna Deva Raya, there were several foreign travelers who visited
the Vijayanagara Empire and left accounts of their experiences. Some of the notable
contemporary foreign travelers of Krishna Deva Raya include Domingo Paes, a
Portuguese traveler who visited the empire in 1522 and left behind an extensive
account of his observations.

• The Portuguese travellers Duarte Barbosa visited Vijayanagara during the time of
Krishna Deva Raya. He described wealth and luxurious life kings and nobles of the
Vijayanagara Empire.

• Fernao Nuniz was a voyager from Portugal. During reign of Achyuta Deva Raya
Portuguese traveller, Fernoa Nuniz visited Vijayanagar Empire. Achyuta Deva Raya was
the younger brother of Krishna Deva Raya. In the years (1535−1537 CE), he was a
chronicler and horse merchant who spent three years in Vijayanagara, the capital of the
Vijayanagara Empire. His writings have revealed numerous fascinating facts about
Vijayanagara at the time.

• Nicolo Dei Conti commonly called Niccolo Conti was an Italian traveller and a merchant
who visited India in 1420 AD at the Vijayanagara empire probably after the accession of
Deva Raya II.

• Abdur Razzak- He was a Persian, Timurid chronicler and a scholar who visited the
Vijaynagar Kingdom at the time of Dev Raya II.

Q.90) Solution (b)

Explanation:

• The term "wali" in Sufism refers to a person who is considered a "friend of God".

• Sufism, also known as tasawuf, encompasses a range of mystical movements and


practices within Islam.
.
IASBABA’S PRELIMS TEST SERIES (AIPTS) 2024 – TEST 5 (MEDIEVAL HISTORY)

• The goal of Sufism is to facilitate a direct connection between humans and God through
personal spiritual experiences grounded in Islamic tradition.

• "Khalifa" refers to a person who succeeds another in a position of leadership or


authority.

• "Silsila" denotes a chain or lineage, particularly within Sufi orders.

• "Ziyarat" refers to a pilgrimage or visitation to a holy site or shrine in Islam.

Q.91) Solution (a)

Explanation:

• Ibrahim Adil Shah II, the fifth ruler of the Adil Shahi dynasty, was a prominent figure
known for his religious tolerance and secular beliefs, earning him the title of Jagadguru
Budshah. He made efforts to foster cultural harmony between various religious
communities, including Shia and Sunni Muslims as well as Hindus, through the medium
of music. (Hence statement 1 is correct).

• Music was one of his passions, and he was known to play instruments, sing, and even
compose songs in praise of Hindu deities such as Saraswati and Ganapati. (Hence
statement 2 is incorrect).

• In fact, he authored a book titled Kitab-E-Navras (Book of Nine Rasas) in Dakhani.The


Sultan was a patron of the arts and maintained a sizable number of musicians,
estimated to be around 4000, at his court. This musical ensemble was referred to as the
Lashkar-e-Nauras (Army of Nauras) and was regularly paid by the government. (Hence
statement 3 is correct).

• Ibrahim Rauza is a mausoleum complex located in Bijapur, Karnataka, India, and it was
built by Ibrahim Adil Shah II as a tomb for himself and his queen, Taj Sultana. The
complex was completed in 1626 and is considered one of the most important examples
of Deccan architecture.

• The complex consists of two main buildings: the tomb of Ibrahim Adil Shah II and the
tomb of Taj Sultana. Both buildings are constructed of black basalt and feature intricate
carvings and inlaid decorations, including Arabic inscriptions and floral motifs. The
buildings are connected by a large entrance arch, which is also adorned with intricate
carvings. It is commonly referred to as the "Taj Mahal of South India".
.
IASBABA’S PRELIMS TEST SERIES (AIPTS) 2024 – TEST 5 (MEDIEVAL HISTORY)

Q.92) Solution (c)

Explanation:

• Gol Gumbaz is a mausoleum located in Bijapur, Karnataka, India, and was built in the
17th century during the reign of the Adil Shahi dynasty.

• It is one of the most famous monuments in Bijapur and a major tourist attraction in the
region.

• The name "Gol Gumbaz" means "circular dome" in Persian, and it is known for its large,
unsupported dome that measures 44 meters in diameter.

• The dome of Gol Gumbaz is supported by eight intersecting arches, and it is said to be
one of the largest of its kind in the world. (Hence statement 1 is correct).

• The central hall of Gol Gumbaz contains the tombs of Muhammad Adil Shah, his wives,
and his daughters. (Hence statement 2 is correct).

• There is also a Whispering Gallery in Gol Gumbaz, which is a circular balcony that runs
around the outside of the dome. The gallery is known for its unique acoustic properties,
where a whisper on one side can be heard on the other side due to the sound waves
bouncing off the dome.

• Gol Gumbaz is considered a masterpiece of Deccan architecture and is known for its
intricate carvings, beautiful calligraphy, and Persian inscriptions.

• The mausoleum is an important cultural and historical monument that reflects the rich
cultural heritage of the Adil Shahi dynasty and the Deccan plateau.

• It has been declared a protected monument by the Archaeological Survey of India and is
open to visitors throughout the year. (Hence statement 3 is correct).

Q.93) Solution (b)

Explanation:

Akbar II, who lived from 1806 to 1837 CE, was under the protection of the British as they had
captured Delhi in 1803 CE. During his reign, he bestowed the title of 'Raja' on Ram Mohan Roy.
.
IASBABA’S PRELIMS TEST SERIES (AIPTS) 2024 – TEST 5 (MEDIEVAL HISTORY)

In addition to being a ruler, Akbar II was also a renowned poet and is known for establishing the
Hindu-Muslim unity festival known as Phool Walon Ki Sair.

Q.94) Solution (a)

Explanation:

• Jahandar Shah was the Mughal Emperor of India who ruled from 1712 to 1713 CE. He
was the son of Bahadur Shah I and was chosen as his successor after his death in 1712.
However, Jahandar Shah was known for his weak leadership and extravagant lifestyle,
which caused discontent among the nobles and the people. He also appointed his
favorites to key positions, which further fueled the dissatisfaction of the nobility.

• Farrukhsiyar was a Mughal Emperor of India who ruled from 1713 to 1719 CE. He was
the grandson of Aurangzeb and succeeded Jahandar Shah to the throne with the
support of the nobles. However, his reign was marked by political instability, rebellions,
and corruption.

• Muhammad Shah Rangeela was a Mughal Emperor of India who ruled from 1719 to
1748 CE. He was the son of Khujista Akhtar, one of the daughters of Emperor Bahadur
Shah I. Muhammad Shah's reign was marked by political instability, weak governance,
and the rise of regional powers.

• Shah Alam II was a Mughal Emperor of India who ruled from 1759 to 1806 CE. He was
the son of Alamgir II and was installed as the emperor by the Maratha Empire after they
defeated the Sayyid brothers, who had deposed his father.

Q.95) Solution (d)

Explanation:

• The Maratha Empire was a powerful Indian dynasty that emerged as a small kingdom in
western India in the 17th century. Over time, the Marathas expanded their territory
through a series of battles and alliances, eventually establishing Raigad as their capital.
(Hence statement 1 is correct).

• One of the key reasons for the Marathas' success was their effective use of guerrilla
warfare tactics. Due to the rugged terrain in which they operated, the Marathas were
.
IASBABA’S PRELIMS TEST SERIES (AIPTS) 2024 – TEST 5 (MEDIEVAL HISTORY)

able to station their soldiers in dispersed sections of villages and launch surprise attacks
on invading forces from hilltop forts. (Hence statement 2 is correct).

• Shivaji was the founder of the Maratha Empire, and he was crowned as Chatrapati
(Sovereign) in 1674. He was a brilliant military strategist and leader who played a
pivotal role in the expansion of the Maratha Empire. Under Shivaji's leadership, the
Marathas established themselves as a formidable force in India, and they continued to
play a prominent role in Indian history well into the 19th century. (Hence statement 3 is
correct).

Q.96) Solution (c)

Explanation:

• Rajendra Chola: He was a great patron of learning and he was known as Pandita-Chola.
He was known as Napoleon of South India due to his military conquest. He was the one
who invaded and conquered Sri Lanka and later Kalinga. (Hence pair 1 is correctly
matched).

• Rajaraja I: The youngest son of Sundara Chola. He was raised in Pazhayarai by Periya
Piratti and Ilaya Piratti. He was taught well by his sister and was sent to Sri Lanka at the
age of 19 for battle. He was said to be saved by Mother Kaveri herself from drowning in
the river when he was 5, earning him the title "Ponniyin Selvan" ("Son of Kaveri").
(Hence pair 2 is correctly matched).

• Rajadhiraja: He was a great warrior who always led from the front standing shoulder to
shoulder with his men on front lines and earned the title of Jayamkonda Chola (the
victorious Chola King). He ably defeated the pandyas, Cheras and the ruler of Srilanka.
(Hence pair 3 is correctly matched).

Q.97) Solution (a)

Explanation:

• Qazi Fazilat (in office: 1541–1545) was the Governor of Bengal. He was appointed by
Sher Shah Suri during his reign in Delhi.

• Qazi Fazilat was also known for his harsh and uncompromising interpretation of Islamic
law, and he implemented strict punishments for crimes such as theft and adultery. He
.
IASBABA’S PRELIMS TEST SERIES (AIPTS) 2024 – TEST 5 (MEDIEVAL HISTORY)

was also involved in the administration of the empire and was responsible for collecting
taxes and dispensing justice.

Q.98) Solution (c)

Explanation:

• The war of succession that took place in 1657-58 A.D. among the sons of Shahjahan was
a major event in Mughal history. After Shahjahan fell ill and was unable to rule, his four
sons, Dara Shikoh, Shuja, Aurangzeb, and Murad, all laid claim to the throne. This
resulted in a bitter power struggle, which lasted for over a year and led to widespread
conflict and bloodshed.

• Dara Shikoh was the eldest of the four brothers and was initially seen as the frontrunner
in the race for the throne. He was a patron of the arts and was known for his liberal
religious views. Shuja, the second-eldest, was the governor of Bengal and had a
reputation for being a skilled military commander. Aurangzeb, the third son, was the
governor of Deccan and was known for his strong religious convictions. Murad, the
youngest, was the governor of Gujarat and was known for his love of luxury and
extravagance.

• During the war of succession, each brother sought to gain the support of the Mughal
nobility, military commanders, and other influential figures. Battles were fought across
the Mughal Empire, and alliances shifted frequently as the brothers jostled for position.

• Ultimately, Aurangzeb emerged as the victor, and he ascended to the Mughal throne in
1658. He went on to become one of the most significant rulers in Indian history and
expanded the Mughal Empire to its greatest extent

Q.99) Solution (b)

Explanation:

• Bhakti movement and the Sufi movement were contemporary to each other. Both
movements emerged around the same time in medieval India, during the 12th to 17th
centuries. They shared some common features, such as a rejection of the rigid caste
system and the formalities of orthodox religion, and emphasized the importance of a
personal, emotional connection with the divine. However, they also had significant
.
IASBABA’S PRELIMS TEST SERIES (AIPTS) 2024 – TEST 5 (MEDIEVAL HISTORY)

differences in terms of their religious beliefs, practices, and the social and cultural
contexts in which they emerged. (Hence statement 1 is correct)

• Bhakti movement emphasized the idea of God's unity or oneness. The movement
promoted the belief that there is only one God who is present in all beings and that
devotion to this God could lead to spiritual liberation. The Bhakti saints rejected the
traditional Hindu caste system and emphasized the importance of a personal
relationship with God, which was accessible to people from all castes and genders.
Through the use of poetry, music, and other forms of artistic expression, they spread
their message of devotion to God and social equality throughout India. (Hence
statement 2 is correct)

Q.100) Solution (b)

Explanation:

• Sher Shah Suri, also known as Farid Khan, founded the Suri Empire in India with its
capital situated in present-day Bihar. (Hence statement 1 is correct).

• He is famous for rebuilding the Grand Trunk Road, which spanned from the Indus River
in the west to Sonargaon in Bengal. (Hence statement 2 is correct).

• Unfortunately, Sher Shah suffered severe injuries during a cannon blast while he was
besieging a fort, and he later passed away in 1545. (Hence statement 3 is incorrect).

Other Reforms of Sher Shah Suri

• Administrative Reforms: Sher Shah Suri implemented significant administrative reforms


that had a lasting impact on governance in the region. He introduced an efficient
revenue system, known as the "Dahsala," which involved a comprehensive survey of
agricultural lands and the imposition of a fixed tax rate. This helped in streamlining
revenue collection and increasing the state's income.

• Road Infrastructure: Sher Shah Suri is renowned for his efforts in developing a vast
network of roads, including the famous Grand Trunk Road (also known as Shahi Road).
This road connected several major cities and towns across the empire, facilitating trade,
communication, and military movements. The road network improved transportation
and boosted economic activities.
.
IASBABA’S PRELIMS TEST SERIES (AIPTS) 2024 – TEST 5 (MEDIEVAL HISTORY)

• Postal System: Sher Shah Suri established an efficient postal system, known as the "Dak
Chauki," which facilitated communication and information exchange across his empire.
The postal system had strategically placed stations where messengers and travelers
could rest, and fresh horses were available for faster communication.

• Currency Reforms: Sher Shah Suri introduced a standard silver coin, known as the
"Rupiya," which became the basis for the modern Indian rupee. This currency reform
helped stabilize the economy and facilitated trade and commerce within the empire.

• Land Reforms: Sher Shah Suri implemented land reforms aimed at protecting the rights
of farmers and ensuring equitable distribution of agricultural lands. He introduced
measures to prevent the exploitation of peasants by local landlords and established a
system of regularizing land ownership.

• Military Reforms: Sher Shah Suri's military reforms strengthened the army and
enhanced its efficiency. He organized a standing army known as the "Tuman," consisting
of well-trained infantry, cavalry, and artillery. He also introduced the use of gunpowder
weapons, including cannons and firearms, in the army.

• Architecture and Construction: Sher Shah Suri was known for his architectural
endeavors. He constructed several buildings and forts, including the famous Rohtas Fort
in present-day Pakistan. The architecture displayed a blend of Afghan, Persian, and
Indian styles.

You might also like